Qui si risolve LOGO
a

Menu

M

Chiudi

Esercizi su punti stazionari con determinante hessiano nullo

Massimi e minimi liberi e vincolati, Metodi matematici per la meccanica classica

Home » Esercizi su punti stazionari con determinante hessiano nullo

Nella ricerca dei punti di massimo e minimo per una funzione derivabile, è molto utile il teorema di Fermat: tali punti di estremo vanno ricercati tra i punti stazionari della funzione, ossia i punti in cui la sua derivata o il suo gradiente si annullano. Non tutti i punti stazionari sono però di massimo o di minimo; inoltre, il teorema non dà alcuna informazione su come distinguere i punti di massimo da quelli di minimo. Anche ai fini delle applicazioni pratiche come la ricerca dei punti di stabilità di un sistema meccanico, risulta di importanza fondamentale classificare quali punti stazionari siano di massimo, di minimo, oppure né di massimo né di minimo. Il segno della derivata seconda della funzione nel punto considerato è un primo modo di procedere. Per funzioni in più variabili, ciò corrisponde a studiare la segnatura della matrice hessiana della funzione, cioè determinarne il segno degli autovalori. Purtroppo tale metodo non è sempre conclusivo, in quanto in presenza di autovalori nulli (cioè se il determinante hessiano è nullo) può capitare di non trarre abbastanza informazioni per stabilire con certezza la natura del punto stazionario considerato.

In questa raccolta di 16 esercizi presentiamo dei casi in cui tale situazione si verifica, mostrando varie tecniche per portare a termine lo studio della natura dei punti stazionari. Gli esercizi sono completamente risolti e ampiamente illustrati, per fornire al lettore spiegazioni chiare e dettagliate. Gli esercizi vanno oltre il semplice studio dei punti stazionari, presentando lo studio completo degli estremi locali e globali delle funzioni in più variabili coinvolte.
Le tecniche qui presentate sono estremamente utili anche agli studenti dei corsi di Meccanica Razionale, in quanto spesso lo studio della stabilità di sistemi meccanici si riconduce a studiare la natura di punti critici in cui appunto il determinante hessiano si annulla. Infatti alcuni dei seguenti esercizi sono proprio tratti da casistiche simili.
L’articolo è quindi uno strumento dedicato a chi desidera approfondire a 360° la propria competenza nello studio delle funzioni in più variabili, fornendo una panoramica completa sull’argomento.

Oltre all’esaustiva lista reperibile in fondo all’articolo, segnaliamo anche le seguenti pagine contenenti materiale affine:

Buona lettura!

 

Sommario

Leggi...

Questa dispensa è una raccolta di esercizi svolti sullo studio dei punti di massimo e minimo assoluti e relativi per funzioni di più variabili, con particolare attenzione al caso in cui la matrice hessiana sia semidefinita ma non definita, che in due variabili è equivalente al fatto che essa abbia determinante nullo. Dopo un breve sunto della teoria, negli esercizi vengono mostrati vari metodi per affrontare tale studio. Segnaliamo che alcuni problemi e parte delle relative soluzioni sono tratti da [6].

 
 

Autori e revisori


 
 

Notazioni

Leggi...

\mathbb{N} Insieme dei numeri naturali;
\mathbb{Z} Insieme dei numeri interi relativi;
\mathbb{R} Insieme dei numeri reali;
\mathbb{R}^n Insieme delle n-uple di numeri reali;
\mathbf{0} Vettore nullo in \mathbb{R}^n;
e_i i-esimo vettore coordinato in \mathbb{R}^n: (0,\dots,0,1,0,\dots,0);
f_{x_i}(\bar{x}), \frac{\partial f}{\partial x_i}(\bar{x}) Derivata parziale di f \colon \mathbb{R}^n \to \mathbb{R} nel punto \bar{x} rispetto alla direzione e_i;
\nabla f(\bar{x}), D f(\bar{x}) Gradiente di f \colon \mathbb{R}^n \to \mathbb{R} nel punto \bar{x};
f_{x_i x_j}(\bar{x}), \dfrac{\partial^2 f}{\partial x_i \partial x_j}(\bar{x}) Derivata seconda di f \colon \mathbb{R}^n \to \mathbb{R} nel punto \bar{x} rispetto alle coordinate x_i e x_j;
\nabla^2 f(\bar{x}), D^2 f(\bar{x}) Matrice hessiana di f \colon \mathbb{R}^n \to \mathbb{R} nel punto \bar{x};
\mathcal{C}^{k}(A) Spazio delle funzioni derivabili k volte in A con derivate continue;
\mathcal{C}^{\infty}(A) Spazio delle funzioni derivabili infinite volte in A con derivate continue;
\mathbb{R}^{m \times n} Spazio vettoriale delle matrici a coefficienti reali aventi m righe e n colonne;
I Matrice identità di dimensione deducibile dal contesto;
\det M Determinante della matrice M \in \mathbb{R}^{n \times n};
\operatorname{Tr} M Traccia della matrice M \in \mathbb{R}^{n \times n}, ossia somma degli elementi sulla diagonale principale di M;
\sup_A f, \inf_A f Estremi inferiore e superiore di f sull’insieme A;
\max_A f, \min_A f Massimo e minimo di f sull’insieme A;
\partial A Frontiera dell’insieme A \subseteq \mathbb{R}^n.


 
 

Introduzione

Leggi...

Lo studio degli estremi relativi e assoluti di una funzione reale assume notevole importanza dal punto di vista pratico, soprattutto in applicazioni di tipo fisico ed economico. In molti problemi, un obiettivo fondamentale consiste infatti nel determinare per quali scelte dei parametri alcune funzioni (che possono modellizzare ad esempio il costo di un progetto o la durata di un componente) assumono valore massimo o minimo. Nei corsi di Analisi Matematica 1, lo studente è diventato familiare con tali problemi di ottimizzazione, nel caso in cui la funzione in esame dipendesse da una sola variabile reale.

Non è difficile fornire esempi (e anzi in natura tali esempi abbondano) di casi in cui le funzioni che si vuole studiare dipendono da più variabili reali. Ad esempio, il costo di trasporto di un determinato bene su un mezzo come un camion dipende sia dalla lunghezza del tratto percorso, ma anche dal carico a cui il mezzo è sottoposto. Oppure, la durata di un componente dipende dal tempo di utilizzo dello stesso, ma anche dallo stress che è costretto a sopportare.

È dunque naturale studiare tecniche per lo studio di problemi di massimo e di minimo anche per funzioni che dipendano da più variabili reali. Tale scopo è uno dei principali dei corsi di Analisi Matematica 2. Come il lettore può immaginare, le derivate costituiscono anche in questo ambito lo strumento principale per affrontare tale studio.

Nel caso di funzioni di una variabile reale, un metodo per determinare estremi locali di una funzione f \colon \mathbb{R} \to \mathbb{R} derivabile 2 volte con continuità consiste nell’usare il teorema di Fermat, che afferma che, se f è derivabile in un punto x_0 di estremo locale, allora f'(x_0)=0. Dunque, per determinare i punti di estremo locale di f, si determinano le soluzioni dell’equazione

(1)   \begin{equation*} f'(x) = 0, \end{equation*}

ossia i cosiddetti punti stazionari di f. Una volta ottenuto un punto stazionario x_0, una conseguenza del teorema di Taylor è la seguente:

    \[\quad\]

  1. se f''(x_0)>0, allora x_0 è un punto di minimo locale per f;
  2.  

  3. se f''(x_0)<0, allora x_0 è un punto di massimo locale per f;
  4.  

  5. se f''(x_0)=0, allora occorre determinare la natura di x_0 in altro modo.

Il criterio esposto non afferma quindi nulla nel caso in cui la derivata seconda di f si annulli: occorre stabilire la natura di x_0 con altri metodi, ad esempio studiando il segno della derivata prima f' in un intorno di x_0, oppure con considerazioni particolari su f (come il suo segno, stime, etc…).

Nel caso di una funzione f \colon \mathbb{R}^n \to \mathbb{R} di più variabili, il processo che solitamente si segue è simile: dal teorema di Fermat 1.9 i punti di estremo locale di f vanno ricercati tra i suoi punti stazionari.

Una volta determinati tali punti stazionari, si può utilizzare il criterio fornito dal teorema 1.10, basato sul “segno delle derivate seconde” di f, analogo di quello per funzioni di una variabile. Purtroppo può accadere, come per il punto 3, che tale criterio non permetta di stabilire la natura del punto stazionario in esame. Ciò avviene quando l’analogo delle derivata seconda in più variabili, detta matrice hessiana di f, è semidefinita ma non definita; ciò, nel caso di funzioni di due variabili, è equivalente al fatto che il determinante della matrice hessiana, detto determinante hessiano di f, si annulli.

In tali casi, occorre stabilire la natura del punto stazionario usando altri metodi. Lo scopo principale di questa dispensa è appunto di analizzare queste situazioni, e fornire al lettore una panoramica varia di strategie per affrontare lo studio della natura dei punti stazionari in cui la matrice hessiana sia semidefinita. Questi metodi si basano principalmente su tre tecniche generali:

    \[\quad\]

  • studio del segno di f (o di una sua variante) in un intorno del punto stazionario \bar{x};
  •  

  • studio del segno delle derivate prime di f in un intorno di \bar{x};
  •  

  • considerazioni e stime particolari su f.

Negli esercizi spiegheremo nel dettaglio queste tecniche e ne forniremo varie applicazioni allo studio dei punti di estremo locale e assoluto per funzioni di più variabili reali. Il lavoro è così organizzato.

    \[\quad\]

  1. Nella sezione 1 ricordiamo brevemente le definizioni e i teoremi principali che utilizzeremo nella risoluzione degli esercizi.
  2.  

  3. Nella sezione 2 riportiamo le tracce degli esercizi che svolgeremo.
  4.  

  5. Nella sezione 3 proponiamo delle soluzioni per gli esercizi. Spesso vengono fornite più soluzioni alternative dei problemi, al fine di permettere al lettore di acquisire varie tecniche e affinare la sua capacità di valutare quale pregio abbia ciascuna di esse.

Osserviamo che gli esercizi, pur essendo principalmente improntati allo studio di punti stazionari con hessiano nullo, presentano inoltre lo studio di punti a hessiano non nullo e la ricerca degli estremi assoluti di f. Abbiamo preferito inserire gli esercizi in questo contesto più generale, anche per fornire al lettore una panoramica più ampia dell’argomento dell’ottimizzazione per funzioni a più variabili.

Osserviamo infine che, per semplicità, ci restringiamo principalmente al caso di funzioni di 2 variabili, ma si può facilmente intuire come ragionamenti del tutto simili si applichino anche al caso di funzioni di 3 o più variabili.


 
 

Richiami di teoria

Leggi...

In questa sezione riportiamo le definizioni e i risultati fondamentali sulle funzioni in più variabili che utilizzeremo nel seguito. Non riportiamo le dimostrazioni dei teoremi enunciati, ma rimandiamo il lettore a [2, capitolo 3] e [3, capitolo 3] per una trattazione completa dell’argomento.

Indicheremo il generico punto di \mathbb{R}^n con \bar{x} e le sue coordinate con \bar{x}=(x_1,\dots,x_n). Poiché spesso lavoreremo in \mathbb{R}^2, indicheremo in tali casi le coordinate dei punti di \mathbb{R}^2 con (x,y).

Definizione 1.1 (derivate parziali). Sia A \subseteq \mathbb{R}^n aperto, sia \bar{x} \coloneqq (x_1,\dots,x_n) \in A, sia f \colon A \to \mathbb{R} e sia i \in \{1,\dots,n\}. Si definisce derivata parziale di f rispetto a x_i in \bar{x} il limite (se esiste)

(2)   \begin{equation*} \begin{split} f_{x_i}(\bar{x}) \coloneqq & \lim_{h \to 0} \frac{f(\bar{x}+he_i) - f(\bar{x})}{h} \\ = & \lim_{h \to 0} \frac{f(x_1,x_2,\dots,x_{i-1},x_i + h,x_{i+1},\dots,x_n) - f(x_1,x_2,\dots,x_{i-1},x_i,x_{i+1},\dots,x_n)}{h}. \end{split} \end{equation*}

Se f_{x_i}(\bar{x}) esiste ed è finita, f si dice derivabile parzialmente rispetto a x_i in \bar{x}.

    \[\quad\]

Le derivate parziali di f vengono anche denotate rispettivamente con il simbolo

(3)   \begin{equation*} \frac{\partial f}{\partial x_i}(\bar{x}). \end{equation*}

Nel caso di \mathbb{R}^2, indicheremo le derivate parziali in (x_0,y_0) \in A con

(4)   \begin{equation*} f_x(x_0,y_0),\quad f_y(x_0,y_0), \qquad \text{oppure con} \qquad \frac{\partial f}{\partial x}(x_0,y_0), \quad \frac{\partial f}{\partial y}(x_0,y_0). \end{equation*}

Definizione 1.2 (gradiente, punti stazionari). Sia A \subseteq \mathbb{R}^n aperto, sia f \colon A \to \mathbb{R} e sia \bar{x} \coloneqq (x_1,\dots,x_n) \in A. Se le n derivate parziali di f in \bar{x} esistono e sono finite, f si dice derivabile in \bar{x} e si definisce gradiente di f in \bar{x} il vettore di \mathbb{R}^n delle derivate parziali in \bar{x}, ovvero

(5)   \begin{equation*} \nabla f(\bar{x}) \coloneqq \left(f_{x_1}(\bar{x}),\dots, f_{x_n}(\bar{x})  \right ). \end{equation*}

Se tutte le derivate parziali di f in \bar{x} sono nulle, ossia se \nabla f(\bar{x}) = \mathbf{0}, allora \bar{x} è detto un punto stazionario di f.

    \[\quad\]

Per il gradiente vengono tipicamente usati anche i simboli

(6)   \begin{equation*} Df(\bar{x}), \qquad \operatorname{grad}f(\bar{x}). \end{equation*}

Se una funzione di più variabili è derivabile in un aperto, ha senso parlare di derivate parziali seconde.

Definizione 1.3 (derivate seconde). Sia A \subseteq \mathbb{R}^n aperto, sia f \colon A \to \mathbb{R} e sia \bar{x} \in A. Se f è derivabile parzialmente rispetto a x_i in A, si definisce derivata parziale seconda di f rispetto a x_i e x_j in \bar{x} la derivata parziale della funzione f_{x_i} \colon A \to \mathbb{R} rispetto a x_j in \bar{x}, ovvero

(7)   \begin{equation*} f_{x_i x_j}(\bar{x}) \coloneqq (f_{x_i})_{x_j}(\bar{x}) = \lim_{h \to 0} \frac{f_{x_i}(\bar{x} + h e_j) - f_{x_i}(\bar{x})}{h}. \end{equation*}

Le derivate seconde parziali f_{x_i x_i} di f si dicono pure, mentre le derivate seconde parziali f_{x_i x_j} di f, con i \neq j, si dicono miste.

    \[\quad\]

Un’altra notazione molto utilizzata per le derivate seconde di f rispettivamente pure e miste è

(8)   \begin{equation*} \frac{\partial^2 f}{\partial x_i^2}(\bar{x}), \qquad \frac{\partial^2 f}{\partial x_i \partial x_j}(\bar{x}). \end{equation*}

In maniera analoga si definiscono le derivate di f di ordine 3 o superiore.

Definizione 1.4 (matrice hessiana). Sia A \subseteq \mathbb{R}^n aperto, sia f \colon A \to \mathbb{R} e sia \bar{x}\in A. Se le derivate seconde f_{x_i x_j}(\bar{x}) esistono e sono finite per ogni i,j \in \{1, \dots,n\}, la matrice n \times n

(9)   \begin{equation*} \nabla^2 f(\bar{x}) \coloneqq \begin{pmatrix}     f_{x_1 x_1}(\overline{x})	&	f_{x_1 x_2}(\overline{x})		&	\cdots 					& 	f_{x_1 x_n}(\overline{x})\\[6pt]     f_{x_2 x_1}(\overline{x})	&	f_{x_2 x_2}(\overline{x})		&	\cdots 					& 	f_{x_2 x_n}(\overline{x})\\[6pt]     \vdots						&	\vdots							&	\ddots					&	\vdots \\[6pt]     f_{x_n x_1}(\overline{x})	&	f_{x_n x_2}(\overline{x})		&	\cdots 					& 	f_{x_n x_n}(\overline{x})     \end{pmatrix} \end{equation*}

delle derivate seconde di f in \bar{x} si dice matrice hessiana di f in \bar{x}

    \[\quad\]

La matrice hessiana di f in \bar{x} viene anche indicata con i simboli

(10)   \begin{equation*} D^2 f(\bar{x}), \qquad \operatorname{Hess} f(\bar{x}). \end{equation*}

Nel caso di una funzione f \colon \mathbb{R}^2 \to \mathbb{R}, la matrice hessiana di f è una matrice 2 \times 2 della forma

(11)   \begin{equation*} \nabla^2 f(x_0,y_0) = \begin{pmatrix} f_{xx}(x_0,y_0) 	&	f_{xy}(x_0,y_0) \\[6pt] f_{yx}(x_0,y_0) 	&	f_{yy}(x_0,y_0) \end{pmatrix}. \end{equation*}

Definizione 1.5 (funzione di classe \mathcal{C}^k). Sia A \subseteq \mathbb{R}^n aperto e sia f \colon A \to \mathbb{R}.

    \[\quad\]

  1. f si dice di classe \mathcal{C}^1(A) se tutte le derivate parziali di f esistono in ogni punto di A e sono funzioni continue.
  2.  

  3. f si dice di classe \mathcal{C}^k(A) se le derivate parziali di ordine k-1 di f sono a loro volta derivabili in ogni punto di A e le loro derivate sono funzioni continue.
  4.  

  5. f si dice di classe \mathcal{C}^\infty(A) se f è di classe \mathcal{C}^k(A) per ogni k \in \mathbb{N}.

    \[\quad\]

Il seguente teorema afferma che, sotto opportune ipotesi di regolarità, la matrice Hessiana è sempre simmetrica. Per una dimostrazione, il lettore può consultare [2, teorema 3.11] oppure [3, sezione 28]

Teorema 1.6 (Schwartz). Sia A \subseteq \mathbb{R}^n aperto, sia f \colon A \to \mathbb{R} una funzione di classe \mathcal{C}^2(A) e sia \bar{x} \in A. Allora per ogni i,j=1,\dots,n si ha

(12)   \begin{equation*} f_{x_i x_j}(\bar{x}) = f_{x_j x_i} (\bar{x}). \end{equation*}

    \[\quad\]

In altre parole, se f è di classe \mathcal{C}^2, allora la matrice Hessiana \nabla^2 f è simmetrica.

Come nel caso delle funzioni di una variabile reale, le derivate permettono di studiare gli estremi locali e assoluti. Ricordiamo quindi la definizione di tali nozioni.

Definizione 1.7 (punti di estremo locale e assoluto). Sia A \subseteq \mathbb{R}^n, sia f \colon A \to \mathbb{R} una funzione e sia \bar{x} \in A.

    \[\quad\]

  • \bar{x} si dice punto di massimo assoluto se

    (13)   \begin{equation*} f(\bar{x}) \geq f(x) \qquad \forall x \in A. \end{equation*}

    In tal caso, il valore f(\bar{x}) viene detto massimo di f in A e viene indicato col simbolo \max_A f.

  •  

  • \bar{x} si dice punto di minimo assoluto se

    (14)   \begin{equation*} f(\bar{x}) \leq f(x) \qquad \forall x \in A. \end{equation*}

    In tal caso, il valore f(\bar{x}) viene detto minimo di f in A e viene indicato col simbolo \min_A f.

  •  

  • \bar{x} si dice punto di massimo locale o relativo se esiste un intorno U di \bar{x} tale che

    (15)   \begin{equation*} f(\bar{x}) \geq f(x) \qquad \forall x \in U. \end{equation*}

  •  

  • \bar{x} si dice punto di minimo locale o relativo se esiste un intorno U di \bar{x} tale che

    (16)   \begin{equation*} f(\bar{x}) \leq f(x) \qquad \forall x \in U. \end{equation*}

    \[\quad\]

Risulta utile il seguente teorema di Weierstrass, che fornisce l’esistenza di massimi e minimi assoluti per funzioni continue su sottoinsiemi compatti1 di \mathbb{R}^n. Rimandiamo a [2, teorema 1.20] e [3, sezione 22] per la dimostrazione.

Teorema 1.8 (Weierstrass). Sia C \subseteq \mathbb{R}^n un insieme compatto e sia f \colon C \to \mathbb{R} una funzione continua. Allora esistono \bar{x}_m, \bar{x}_M \in C punti rispettivamente di minimo e massimo assoluto per f.

    \[\quad\]

Anche per funzioni di più variabili vale l’analogo del teorema di Fermat valido per funzioni di una variabile [4, teorema 1]: un punto di estremo locale in cui la funzione è derivabile è anche un punto stazionario, ovvero in cui tutte le derivate della funzione si annullano. Il lettore può consultare [2, teorema 3.12] oppure [3, sezione 37] per la dimostrazione.

Teorema 1.9 (Fermat). Sia A \subseteq \mathbb{R}^n aperto, sia f \colon A \to \mathbb{R} una funzione e sia \bar{x} \in A. Se \bar{x} è un punto di estremo locale e f è derivabile in \bar{x}, allora \nabla f(\bar{x}) = \bar{0}, ovvero \bar{x} è un punto stazionario di f.

    \[\quad\]

Tale teorema afferma dunque che, per una funzione ovunque derivabile, i punti di estremo locale e assoluto vanno cercati tra i suoi punti stazionari. Esso suggerisce pertanto un metodo per determinare i punti di estremo locale di una funzione f \colon A \to \mathbb{R} con A \subseteq \mathbb{R}^n aperto:

    \[\quad\]

  1. Si verifica che la funzione è ovunque derivabile;
  2.  

  3. Si determinano i punti stazionari di f, cioè si risolve il sistema

    (17)   \begin{equation*} \nabla f(x_1,\dots,x_n) = (0,\dots,0). \end{equation*}

  4.  

  5. Si cerca di stabilire se i punti stazionari trovati siano o meno punti di estremo locale per f.

L’ultimo punto del metodo sopra esposto è essenziale, in quanto la condizione di essere stazionario è necessaria ma non sufficiente affinché il punto sia di estremo locale. Il prossimo criterio permette, in alcuni casi, di determinare la natura del punto stazionario. In altri casi, esso fornisce soltanto delle informazioni parziali, e la classificazione va quindi affrontata con argomenti di tipo diverso, come vedremo nel corso degli esercizi. Per una dimostrazione del teorema, si consulti [2, proposizione 3.13] oppure [3, sezione 37].

Teorema 1.10 (classificazione dei punti stazionari). Sia A \subseteq \mathbb{R}^n aperto, sia f \colon A \to \mathbb{R} una funzione di classe \mathcal{C}^2(A) e sia \bar{x} \in A un punto stazionario di f. Allora valgono le seguenti implicazioni:

    \[\quad\]

  1. se \nabla^2 f(\bar{x}) è definita positiva, allora \bar{x} è un punto di minimo locale per f;
  2.  

  3. se \nabla^2 f(\bar{x}) è definita negativa, allora \bar{x} è un punto di massimo locale per f;
  4.  

  5. se \nabla^2 f(\bar{x}) è indefinita, allora \bar{x} è un punto di sella per f;

Nel caso in cui la matrice hessiana \nabla^2 f(\bar{x}) sia semidefinita, ossia possieda degli autovalori nulli, il criterio di sopra non permette di classificare univocamente \bar{x}, ma solo di escludere alcuni casi. Infatti:

    \[\quad\]

  1. se \nabla^2 f(\bar{x}) è semidefinita positiva e non nulla, allora \bar{x} è un punto di minimo locale o un punto di sella per f;
  2.  

  3. se \nabla^2 f(\bar{x}) è semidefinita negativa e non nulla, allora \bar{x} è un punto di massimo locale o un punto di sella per f.

    \[\quad\]

Ricordiamo al lettore che una matrice simmetrica reale M \in \mathbb{R}^{n \times n} (quale è la matrice hessiana di una funzione di classe \mathcal{C}^2, come stabilito dal teorema 1.6) è sempre diagonalizzabile per il teorema spettrale [1, teorema 15.8]. Dunque vale la seguente definizione/caratterizzazione.

    \[\quad\]

  • M si dice definita positiva se v^T M v >0 per ogni v \in \mathbb{R}^n \setminus \{\mathbf{0}\}. Ciò avviene se e solo se gli autovalori di M sono tutti positivi.
  •  

  • M si dice definita negativa se v^T M v <0 per ogni v \in \mathbb{R}^n \setminus \{\mathbf{0}\}. Ciò avviene se e solo se gli autovalori di M sono tutti negativi.
  •  

  • M si dice indefinita se v^T M v non assume sempre lo stesso segno al variare di v \in \mathbb{R}^n \setminus \{\mathbf{0}\}. Ciò avviene se e solo se M possiede autovalori di segno discorde.
  •  

  • M si dice semidefinita positiva se v^T M v \geq 0 per ogni v \in \mathbb{R}^n. Ciò avviene se e solo se gli autovalori di M sono tutti non-negativi.
  •  

  • M si dice semidefinita negativa se v^T M v \leq 0 per ogni v \in \mathbb{R}^n. Ciò avviene se e solo se gli autovalori di M sono tutti non-positivi.

Pertanto, per utilizzare il criterio fornito dal teorema 1.10, occorre determinare i segni degli autovalori di \nabla^2 f calcolata nei punti stazionari. A tal fine, risultano utili i seguenti criteri. Il primo è detto criterio di Cartesio e si basa sul determinare i segni delle radici del polinomio caratteristico p(\lambda) di M considerando le variazioni di segno dei coefficienti di p(\lambda).

Teorema 1.11 (criterio di Cartesio, [1, teorema 16.4]). Sia M \in \mathbb{R}^{n \times n} una matrice simmetrica e sia p(\lambda)=a_n \lambda^n + \dots + a_d \lambda^d il suo polinomio caratteristico, dove d \in \{0,\dots,n\} e a_d \neq 0. Allora:

    \[\quad\]

  1. la molteplicità dell’autovalore nullo per M è pari a d, ossia alla molteplicità algebrica di 0 come radice di p(\lambda);
  2.  

  3. il numero di autovalori positivi di M, contati con la relativa molteplicità, è pari al numero delle variazioni di segno nella successione

    (18)   \begin{equation*} a_n, a_{n-1}, \dots, a_d \end{equation*}

    dei coefficienti non nulli di p.

    \[\quad\]

Il secondo criterio per studiare i segni degli autovalori di una matrice quadrata M \in \mathbb{R}^{n \times n} considera i determinanti delle sottomatrici ottenute dalle prime k righe e k colonne di M, ed è detto criterio di Sylvester.

Teorema 1.12 (criterio di Sylvester, [7]). Sia M \in \mathbb{R}^{n \times n} una matrice simmetrica reale e sia, per ogni k \in \{1,\dots,n\}, M_k \in \mathbb{R}^{k \times k} la matrice ottenuta dalle prime k righe e k colonne di M. Allora le seguenti condizioni sono equivalenti:

    \[\quad\]

  1. M è definita positiva;
  2.  

  3. \det M_k >0 per ogni k \in \{1,\dots,n\}.

Analogamente, M è definita negativa se e solo se \det M_k < 0 per gli indici k \in \{1,\dots,n\} dispari, mentre \det M_k > 0 per gli indici k \in \{1,\dots,n\} pari.

    \[\quad\]

Nel caso in cui M sia una matrice simmetrica 2\times 2, si può studiare la segnatura2 di M nel seguente modo, che utilizzeremo nel corso della dispensa.

Proposizione 1.13 (autovalori di una matrice 2 \times 2). Sia M \in \mathbb{R}^{2 \times 2} una matrice simmetrica reale e siano \det M e \operatorname{Tr} M rispettivamente il determinante di M e la traccia di M. Valgono le seguenti equivalenze.

    \[\quad\]

  1. M è definita positiva se e solo se \det M>0 e \operatorname{Tr}M >0.
  2.  

  3. M è definita negativa se e solo se \det M>0 e \operatorname{Tr}M <0.
  4.  

  5. M è indefinita se e solo se \det M<0.
  6.  

  7. M è semidefinita positiva se e solo se \det M \geq 0 e \operatorname{Tr}M \geq 0.
  8.  

  9. M è semidefinita negativa se e solo se \det M \geq 0 e \operatorname{Tr}M \leq 0.

    \[\quad\]

Dimostrazione. Sia

(19)   \begin{equation*} M = \begin{pmatrix} a_{11} 	& a_{12} \\ a_{12} 	& a_{22} \end{pmatrix} \in \mathbb{R}^{2 \times 2} \end{equation*}

una matrice simmetrica reale. Osserviamo che

(20)   \begin{equation*} p(\lambda) = \det(M- \lambda I) = \det \begin{pmatrix} a_{11} - \lambda 	& a_{12} \\ a_{12} 	& a_{22} - \lambda \end{pmatrix} = \lambda^2 - (\operatorname{Tr} M) \lambda + \det M. \end{equation*}

D’altra parte, poiché M è diagonalizzabile per il teorema spettrale [1, teorema 15.8], sappiamo che p(\lambda) = \det (M - \lambda I) è decomponibile nel prodotto di polinomi di grado 1

(21)   \begin{equation*} p(\lambda) = (\lambda - \lambda_1)(\lambda - \lambda_2) = \lambda^2 - (\lambda_1 + \lambda_2) \lambda + \lambda_1 \lambda_2 \end{equation*}

con \lambda_1, \lambda_2 autovalori di M. Confrontando (20) e (21) otteniamo che

(22)   \begin{equation*} \lambda_1 + \lambda_2 = \operatorname{Tr}M, \qquad \lambda_1 \lambda_2 = \det M. \end{equation*}

Da queste equazioni si ottiene facilmente la conclusione:

    \[\quad\]

  • M è definita positiva se e solo se \lambda_1 e \lambda_2 hanno entrambi segno positivo, ossia se e solo se si ha \det M = \lambda_1 \lambda_2 > 0 e \operatorname{Tr}M = \lambda_1 + \lambda_2 >0;
  •  

  • M è definita negativa se e solo se \lambda_1 e \lambda_2 hanno entrambi segno negativo, ossia se e solo se si ha \det M = \lambda_1 \lambda_2 > 0 e \operatorname{Tr}M = \lambda_1 + \lambda_2 <0;
  •  

  • M è indefinita se e solo se \lambda_1 e \lambda_2 hanno segno discorde e sono non nulli, ossia se e solo se \det M = \lambda_1 \lambda_2 < 0;
  •  

  • M è semidefinita positiva se e solo se \lambda_1 e \lambda_2 sono entrambi non-negativi, ossia se e solo se si ha \det M = \lambda_1 \lambda_2 \geq 0 e \operatorname{Tr}M = \lambda_1 + \lambda_2 \geq 0;
  •  

  • M è semidefinita negativa se e solo se \lambda_1 e \lambda_2 sono entrambi non-positivi, ossia se e solo se si ha \det M = \lambda_1 \lambda_2 \geq 0 e \operatorname{Tr}M = \lambda_1 + \lambda_2 \leq 0.

Osservazione 1.14. In generale, se M \in \mathbb{R}^{n \times n} è una matrice simmetrica e il suo polinomio caratteristico è p(\lambda) = \lambda^n + b_{n-1} \lambda^{n-1} + \dots + b_0, allora con le stesse considerazioni si ottiene che

(23)   \begin{equation*} b_{n-1} = (-1)^{n-1} \operatorname{Tr} M, \qquad b_0 = (-1)^n \det M. \end{equation*}

Dato che il coefficiente b_{n-1} di un polinomio di grado n è pari alla somma delle radici moltiplicata per (-1)^{n-1}, mentre b_0 è pari al prodotto delle radici per (-1)^n, si ha

(24)   \begin{equation*} \operatorname{Tr} M = \lambda_1 + \dots + \lambda_n, \qquad \det M = \lambda_1 \cdots \lambda_n, \end{equation*}

dove \lambda_1, \dots, \lambda_n sono gli autovalori di M ripetuti in base alla loro molteplicità.    


  1. Per il teorema di Heine-Borel [3, sezione 22] essi sono tutti e soli i sottoinsiemi chiusi e limitati di \mathbb{R}^n.
  2.  

    1. ossia i segni degli autovalori di M.

 
 

Testi degli esercizi

Esercizio 1  (\bigstar\bigstar\bigstar\largewhitestar\largewhitestar). Determinare estremi assoluti e locali della funzione f \colon \mathbb{R}^2 \to \mathbb{R} definita da

(25)   \begin{equation*} f(x,y) = (x^2-y-1)(1-x^2-y^2) \qquad \forall (x,y) \in \mathbb{R}^2. \end{equation*}

Svolgimento.

Osserviamo che la funzione è pari rispetto alla variabile x, quindi è sufficiente considerarne il comportamento nel semipiano

(26)   \begin{equation*} H \coloneqq \{(x,y) \in \mathbb{R}^2 \colon x \geq 0\} \end{equation*}

e poi estendere per simmetria i risultati ottenuti.

Notiamo subito che

(27)   \begin{equation*} \sup_{\mathbb{R}^2} f = + \infty, \qquad \inf_{\mathbb{R}^2} f = - \infty. \end{equation*}

Infatti, restringendo f alla retta di equazione x=0, si ha

(28)   \begin{equation*} \lim_{y \to - \infty} f(0,y) = \lim_{y \to - \infty} (-y-1)(1-y^2) = - \infty, \qquad \lim_{y \to + \infty} f(0,y) = \lim_{y \to + \infty} (-y-1)(1-y^2) = + \infty. \end{equation*}

Per determinare gli estremi locali di f, osserviamo che essa è di classe \mathcal{C}^{\infty}(\mathbb{R}^2) poiché somma e prodotto di polinomi. Dunque, per il teorema 1.9, gli estremi locali di f vanno ricercati tra i suoi punti stazionari. Il seguito dell’esercizio è quindi dedicato a determinare e studiare tali punti stazionari.

Svolgendo i prodotti si ha

(29)   \begin{equation*} \begin{split} f(x,y) = & x^2 - x^4 - x^2y^2 - y + x^2y + y^3 - 1 + x^2 + y^2 \\ = & 2x^2 -  x^4 - x^2y^2 - y + x^2y + y^3 - 1 + y^2 \qquad \forall (x,y) \in \mathbb{R}^2. \end{split} \end{equation*}

Calcoliamo ora le derivate parziali di f:

(30)   \begin{equation*} \begin{gathered} \frac{\partial f}{\partial x}(x,y) = 4x - 4x^3 -2xy^2 + 2xy = 2x(2 - 2x^2 -y^2 +y) \qquad \forall (x,y) \in \mathbb{R}^2, \\ \frac{\partial f}{\partial y}(x,y) = -2x^2y - 1 +x^2 +3y^2 +2y \qquad \forall (x,y) \in \mathbb{R}^2, \end{gathered} \end{equation*}

Da ciò segue che

(31)   \begin{equation*} \begin{split} \nabla f(x,y) = (0,0) \iff & \begin{cases} 2x(2 - 2x^2 -y^2 +y) = 0\\ -2x^2y - 1 +x^2 +3y^2 +2y = 0 \end{cases} \\ \iff & \begin{cases} x = 0\\ 3y^2 +2y -1= 0 \end{cases} \vee \begin{cases} 2x^2 = 2  -y^2 +y\\ -4x^2y +2x^2 +6y^2 +4y -2 = 0 \end{cases} \\ \iff & \begin{cases} x = 0\\ (3y-1)(y+1) =0 \end{cases} \vee \begin{cases} 2x^2 = 2  -y^2 +y\\ 2x^2(1-2y) +6y^2 +4y -2 = 0 \end{cases} \end{split} \end{equation*}

    \[\quad\]

  1. Le soluzioni del primo sistema sono

    (32)   \begin{equation*} (x,y) \in  \left \{ \left ( 0, \frac{1}{3} \right ), \left (0,-1\right ) \right \}. \end{equation*}

  2.  

  3. Per il secondo sistema, sostituendo la prima equazione nella seconda si ottengono i sistemi equivalenti

    (33)   \begin{equation*} \begin{split} \begin{cases} 2x^2 = 2  -y^2 +y\\ (2  -y^2 +y)(1-2y) +6y^2 +4y -2 = 0 \end{cases} \iff & \begin{cases} 2x^2 = 2  -y^2 +y\\ 2y^3 +3y^2 + y = 0 \end{cases} \\ \iff & \begin{cases} 2x^2 = 2  -y^2 +y = 0\\[5pt] y \left ( y+\frac{1}{2} \right ) (y+1) = 0. \end{cases} \end{split} \end{equation*}

    Poiché le soluzioni della seconda equazione sono y=0, y=-\dfrac{1}{2}, y=-1, sostituendo nella prima equazione si ottengono tutte le soluzioni del sistema in H:

    (34)   \begin{equation*} (x,y) \in  \left \{ (1,0), \left ( \sqrt{\frac{5}{8}} ,-\frac{1}{2}\right ), (0,-1) \right \} \end{equation*}

Unendo le soluzioni dei due sistemi, si ottiene l’insieme dei punti stazionari di f in H:

(35)   \begin{equation*} \left\{ \left ( 0, \frac{1}{3} \right ), (0,-1), (1,0), \left ( \sqrt{\frac{5}{8}} ,-\frac{1}{2}\right) \right\}. \end{equation*}

Per determinarne la natura, proviamo a utilizzare il teorema 1.10 e calcoliamo le derivate seconde di f.

(36)   \begin{equation*} \begin{gathered} f_{xx}(x,y) = 4 - 12x^2 -2y^2 +2y \qquad \forall (x,y) \in \mathbb{R}^2, \\ f_{xy}(x,y) = f_{yx}(x,y) = -4xy +2x \qquad \forall (x,y) \in \mathbb{R}^2, \\ f_{yy}(x,y) = -2x^2 + 6y +2 \qquad \forall (x,y) \in \mathbb{R}^2. \end{gathered} \end{equation*}

Ricordando la parità di f rispetto a x, studiamo ognuno dei suoi punti stazionari.

\bullet \left (0,\dfrac{1}{3}\right ). La matrice hessiana di f in \left (0,\frac{1}{3}\right ) è pari a

(37)   \begin{equation*} \nabla^2 f\left (0,\frac{1}{3}\right ) = \begin{pmatrix} 4 - \dfrac{2}{9} + \dfrac{2}{3}		&	0 \\[9pt] 0									&	4 \end{pmatrix}, \end{equation*}

che è definita positiva in quanto è diagonale e i suoi autovalori sono gli elementi sulla diagonale principale, che sono entrambi positivi. Da ciò e dal teorema 1.10 segue che \left (0,\dfrac{1}{3}\right ) è un punto di minimo locale.

\bullet \left (0,-1\right ). Vale

(38)   \begin{equation*} \nabla^2 f\left (0,-1\right ) = \begin{pmatrix} 0			&	0 \\[5pt] 0			&	-4 \end{pmatrix}, \end{equation*}

che risulta semidefinita negativa poiché è diagonale e possiede un autovalore nullo e uno negativo. Dunque per il teorema 1.10 (0,-1) è un punto di massimo locale o di sella per f. Per stabilirlo, studiamo il segno di f. Dall’espressione di f e dalla figura 1 si vede che f(0,-1)=0 e che in ogni intorno di (0,-1) f assume segni sia positivi che negativi, infatti:

    \[\quad\]

  • f(0,y)<0 per y<-1;
  •  

  • f(x,y)>0 per -\sqrt{1-x^2} <y<x^2-1 e 0<x<1.

Quindi (0,-1) è di sella per f.

    \[\quad\]

    \[\quad\]

Figura 1: segni assunti dalla funzione f. In rosso le aree in cui f assume segno positivo, in blu le aree dove f assume segno negativo. Poiché nell’intorno di ognuno dei punti (\pm 1,0) e (0,-1) f assume segni opposti, essi sono di sella per f.

    \[\quad\]

    \[\quad\]

\bullet (1,0)

(39)   \begin{equation*} \nabla^2 f(1,0) = \begin{pmatrix} -8			&	2 \\[5pt] 2			&	0 \end{pmatrix} \quad \Longrightarrow \quad \det \nabla^2 f(1,0) = -4, \end{equation*}

quindi \nabla^2 f(1,0) è indefinita per la proposizione 1.13, poiché possiede autovalori non nulli di segno opposto. Da ciò e dal teorema 1.10 segue che (1,0) è un punto di sella per f. Questo poteva essere dedotto anche dallo studio del segno di f mostrato in figura 1. Per la parità di f rispetto alla variabile x, anche (-1,0) è un punto di sella per f.

\bullet \left (\sqrt{\dfrac{5}{8}} ,-\dfrac{1}{2}\right). Abbiamo

(40)   \begin{equation*} \nabla^2 f\left (\sqrt{\frac{5}{8}} ,-\frac{1}{2}\right) = \begin{pmatrix} -5					&	\sqrt{10} \\[5pt] \sqrt{10}			&	-\dfrac{9}{4} \end{pmatrix} \quad \Longrightarrow \quad \det \nabla^2 f\left (\sqrt{\frac{5}{8}} ,-\frac{1}{2}\right) = \frac{45}{4} - 10 = \frac{5}{4} >0, \end{equation*}

da cui segue che gli autovalori di \nabla^2 f\left (\sqrt{\dfrac{5}{8}} ,-\dfrac{1}{2}\right) hanno lo stesso segno, quindi f è definita positiva oppure negativa. Poiché \operatorname{Tr} \nabla^2 f\left (\sqrt{\dfrac{5}{8}} ,-\dfrac{1}{2}\right) < 0, la proposizione 1.13 implica che gli autovalori di \nabla^2 f\left (\sqrt{\dfrac{5}{8}} ,-\dfrac{1}{2}\right) sono entrambi negativi, e quindi essa è definita negativa. Pertanto \left (\pm \sqrt{\dfrac{5}{8}} ,-\dfrac{1}{2}\right) è, per il teorema 1.10, un punto di massimo locale per f. Di nuovo per la parità di f rispetto a x, anche \left (-\sqrt{\dfrac{5}{8}} ,-\dfrac{1}{2}\right) è di massimo locale.


 

Esercizio 2  (\bigstar\bigstar\bigstar\largewhitestar\largewhitestar). Determinare estremi assoluti e locali della funzione f \colon \mathbb{R}^2 \to \mathbb{R} definita da

(41)   \begin{equation*} f(x,y) = x^4 - xy^2 + y^2 \qquad \forall (x,y) \in \mathbb{R}^2. \end{equation*}

Svolgimento.

Osserviamo che f è pari rispetto alla variabile y, quindi è sufficiente studiare il problema nel semipiano

(42)   \begin{equation*} H \coloneqq \{(x,y) \in \mathbb{R}^2\colon y \geq 0\} \end{equation*}

Affermiamo subito che vale

(43)   \begin{equation*} \sup_{\mathbb{R}^2} f = + \infty, \qquad \inf_{\mathbb{R}^2} f = - \infty. \end{equation*}

Infatti, restringendo f alla di equazione y=0, si ottiene

(44)   \begin{equation*} \lim_{x \to \pm \infty} f(x,0) = \lim_{x \to \pm \infty} x^4 = + \infty, \end{equation*}

mentre restringendo f alla parabola di equazione y=x^2 si ha

(45)   \begin{equation*} \lim_{x \to + \infty} f(x,x^2) = \lim_{x \to + \infty} \left( x^4 - x^5 + x^4 \right ) = -\infty. \end{equation*}

Poiché f è di classe \mathcal{C}^{\infty}(\mathbb{R}^2), dal teorema 1.9 segue che i punti di estremo locale per f vanno ricercati tra i suoi punti stazionari. Calcoliamo quindi le derivate parziali di f e determiniamo i punti in cui esse si annullano. Vale

(46)   \begin{gather*} f_x(x,y) = 4x^3-y^2 \qquad \forall (x,y) \in \mathbb{R}^2, \\ f_y(x,y) = -2xy + 2y \qquad \forall (x,y) \in \mathbb{R}^2. \end{gather*}

Da ciò segue che

(47)   \begin{equation*} \begin{split} \nabla f(x,y) = (0,0) \iff & \begin{cases} 4x^3-y^2 = 0\\ -2xy + 2y = 0 \end{cases} \\ \iff & \begin{cases} 4x^3-y^2 = 0\\ 2y(1-x) = 0 \end{cases} \\ \iff & \begin{cases} 4x^3 = y^2\\ y = 0 \end{cases} \vee \begin{cases} 4x^3 = y^2\\ x = 1. \end{cases} \end{split} \end{equation*}

Pertanto i punti stazionari di f in H sono

(48)   \begin{equation*} (0,0), \quad (1,2). \end{equation*}

Per determinarne la natura, proviamo a utilizzare il teorema 1.10 e calcoliamo le derivate seconde di f:

(49)   \begin{gather*} f_{xx}(x,y) = 12x^2 \qquad \forall (x,y) \in \mathbb{R}^2, \\ f_{xy}(x,y) = f_{yx}(x,y) = -2y \qquad \forall (x,y) \in \mathbb{R}^2, \\ f_{yy}(x,y) = 2-2x \qquad \forall (x,y) \in \mathbb{R}^2, \end{gather*}

\bullet (0,0). La matrice hessiana di f in (0,0) è pari a

(50)   \begin{equation*} \nabla^2 f(0,0) = \begin{pmatrix} 0	&	0 \\ 0	&	2 \end{pmatrix}, \end{equation*}

che è semidefinita positiva in quanto è diagonale e possiede un autovalore nullo e uno positivo. Dunque dal teorema 1.10 segue che esso è di minimo locale o di sella per f. Mostriamo alcuni metodi per stabilire quindi in quale dei due casi (0,0) ricada.

  • ✦ Si ha

    (51)   \begin{equation*} f(x,y) = x^4 + y^2 (1-x) \geq 0 \qquad \forall (x,y) \colon x\leq 1, \end{equation*}

    e quindi, poiché f(0,0)=0, tale punto è di minimo relativo per f.

  •  

  • ✦ Alternativamente, si può ottenere lo stesso risultato studiando il segno delle derivate parziali prime di f in un intorno di (0,0). Si fissi (x_0,y_0) con x_0 < 1 e y_0 \geq 0; supponiamo che x_0 \geq 0, in quanto l’altro caso è analogo. Si ha

    (52)   \begin{equation*} f_x(x,0) = 4x^3 > 0 \qquad \forall x> 0, \end{equation*}

quindi la restrizione di f alla retta y=0 è strettamente crescente nel verso delle x positive, situazione rappresentata nella figura 2, in cui la monotonia di f è rappresentata dalle frecce, che puntano nella direzione in cui essa è crescente.

    \[\quad\]

    \[\quad\]

Figura 2: monotonia di f se ristretta alle rette di equazione y=0 e x=x_0. Le frecce indicano la direzione in cui f è crescente ottenuta studiando rispettivamente i segni di f_x e f_y. Da ciò segue che (0,0) è un punto di minimo relativo per f.

    \[\quad\]

    \[\quad\]

Da ciò segue che f(x_0,0) \geq f(0,0).

Inoltre si ha

(53)   \begin{equation*} f_y(x_0,y) = 2y(1-x_0) > 0 \qquad \forall y > 0, \end{equation*}

in quanto x_0<1, da cui segue che la restrizione di f alla retta di equazione x=x_0 è strettamente crescente nel verso delle y positive, di nuovo rappresentato con le frecce in figura 2. Quindi abbiamo

(54)   \begin{equation*} f(x_0,y_0) > f(x_0,0) > f(0,0). \end{equation*}

Poiché y_0>0 e 0 \leq x_0<1 sono arbitrari, ciò mostra che (0,0) è un punto di minimo relativo per f.

\bullet (1,2). La matrice hessiana di f in (1,2) è pari a

(55)   \begin{equation*} \nabla^2 f(1,2) = \begin{pmatrix} 12		&	- 4 \\ - 4	&	0 \end{pmatrix} \quad \Longrightarrow \quad \det \nabla^2 f(1,2) = -16<0, \end{equation*}

quindi essa è indefinita per la proposizione 1.13 e pertanto il punto (1, 2) risulta di sella per f per il teorema 1.10. Per la parità di f rispetto a y, anche il punto (1,-2) è di sella per f.


 

Esercizio 3  (\bigstar\bigstar\bigstar\bigstar\largewhitestar). Determinare estremi assoluti e locali della funzione f \colon \mathbb{R}^2 \to \mathbb{R} definita da

(56)   \begin{equation*} f(x,y) = x e^y - y e^x \qquad \forall (x,y) \in \mathbb{R}^2. \end{equation*}

Svolgimento.

Osserviamo innanzitutto che f \in \mathcal{C}^{\infty}(\mathbb{R}^2) in quanto somma e prodotto di funzioni derivabili infinite volte con continuità. Vale

(57)   \begin{equation*} \sup_{\mathbb{R}^2} f = +\infty, \qquad \inf_{\mathbb{R}^2} f = -\infty. \end{equation*}

Infatti, restringendoci alla retta di equazione y=0, si ha

(58)   \begin{equation*} \lim_{x \to \pm \infty} f(x,0) = \lim_{x \to \pm \infty} x = \pm \infty. \end{equation*}

Per determinare eventuali punti di estremo locale per f, calcoliamo le derivate parziali di f:

(59)   \begin{equation*} \begin{gathered} f_x(x,y) = e^y - ye^x \qquad \forall (x,y) \in \mathbb{R}^2, \\ f_y(x,y) = xe^y-e^x \qquad \forall (x,y) \in \mathbb{R}^2. \end{gathered} \end{equation*}

Usiamo il teorema 1.9 e ricerchiamo eventuali estremi locali tra i punti stazionari di f. Mostriamo due metodi per studiare tali punti stazionari e la loro natura.

    \[\quad\]

  • Come primo metodo, determiniamo la natura dei punti stazionari senza calcolarli esplicitamente. Affermiamo infatti che qualsiasi punto stazionario è un punto di sella per f. Per convincersi di tale affermazione, assumiamo che (x_0,y_0) sia un punto stazionario per f. Osservando (59), dall’espressione di f_x segue che y_0>0, altrimenti f_x(x_0,y_0) sarebbe pari alla somma di un termine positivo e uno non-negativo, quindi non potrebbe annullarsi. Analogamente, dall’espressione di f_y segue che x_0>0.

    Osserviamo ora che

    (60)   \begin{equation*} f_x(x,y_0) = e^{y_0} - y_0 e^x < e^{y_0} - y_0 e^{x_0} = 0 \qquad \forall x > x_0, \end{equation*}

    dove la disuguaglianza stretta segue da y_0>0 e dal fatto che la funzione x \mapsto e^x è strettamente crescente.

    Questa condizione sul segno di f_x assicura che la restrizione di f alla retta y=y_0 è strettamente decrescente per x > x_0. Ciò è rappresentato in figura 3 con le frecce indicate sulla retta orizzontale di equazione y=y_0, che puntano nel verso in cui f cresce, ossia verso sinistra.

    Da questa monotonia segue che

    (61)   \begin{equation*} f(x,y_0) < f(x_0,y_0) \qquad \forall x > x_0. \end{equation*}

        \[\quad\]

        \[\quad\]

    Figura 3: rappresentazione dello studio delle derivate parziali sulle rette di equazione y=y_0 e x=x_0. Le frecce indicano la direzione in cui f è crescente ottenuta studiando rispettivamente i segni di f_x e f_y. Da (61) si ottiene che f è decrescente per x>x_0 sulla retta y=y_0. Mentre da (62) si vede che f è crescente per y>y_0. Da ciò segue che l’eventuale punto stazionario (x_0,y_0) è di sella.

        \[\quad\]

        \[\quad\]

    Applicando un ragionamento del tutto analogo a f_y, si vede

    (62)   \begin{equation*} f(x_0,y) > f(x_0,y_0) \qquad \forall y > y_0, \end{equation*}

    situazione rappresentata dalle frecce sulla retta verticale di equazione x=x_0 in figura 3. Confrontando (61) e (62), si vede che, in ogni intorno di (x_0,y_0), f assume sia valori maggiori che minori di f(x_0,y_0) e quindi tale punto non può essere né di massimo né di minimo locale. Esso è dunque un punto di sella.

  •  

  • Mostriamo ora il metodo che consiste nel determinare esplicitamente i punti stazionari di f e poi studiarne la natura usando le derivate parziali seconde di f. Considerando (59), si ha

    (63)   \begin{equation*} \begin{split} \nabla f(x,y) = (0,0) \iff & \begin{cases} e^y - ye^x = 0\\ xe^y-e^x = 0 \end{cases} \\ \iff & \begin{cases} e^y - yx e^y = 0\\ e^x = xe^y \end{cases} \\ \iff & \begin{cases} y=\dfrac{1}{x}\\[6pt] e^x = x e^{\frac{1}{x}} \end{cases} \end{split} \end{equation*}

    Dunque per determinare i punti stazionari di f occorre risolvere l’equazione e^x=x e^{\frac{1}{x}}. Osserviamo innanzitutto che essa non possiede soluzioni negative, in quanto il primo membro è sempre positivo e che possiede la soluzione banale x=1. Prendendo il logaritmo naturale di entrambi i membri, il problema diventa equivalente a studiare le soluzioni di

    (64)   \begin{equation*} x = \log x + \frac{1}{x}, \end{equation*}

    ovvero gli zeri della funzione g \colon (0,+\infty) \to \mathbb{R} definita da

    (65)   \begin{equation*} g(x) = x - \log x - \frac{1}{x} \qquad \forall x >0. \end{equation*}

    Si ha

    (66)   \begin{equation*} g'(x) = 1 - \frac{1}{x} + \frac{1}{x^2} = \frac{x^2 - x + 1}{x^2} >0 \qquad \forall x >0, \end{equation*}

    Abbiamo già notato che x=1 è uno zero di g. Poiché g è strettamente crescente, x=1 è l’unico zero di g, che è quindi l’unica soluzione per l’equazione e^x=x e^{\frac{1}{x}}.

    Pertanto l’unico punto stazionario di f è dato da

    (67)   \begin{equation*} (1,1). \end{equation*}

    Calcoliamo le derivate seconde di f al fine di applicare il teorema 1.10:

    (68)   \begin{gather*} f_{xx}(x,y) = -ye^x \qquad \forall (x,y) \in \mathbb{R}^2, \\ f_{xy}(x,y)= f_{yx}(x,y) = e^y - e^x \qquad \forall (x,y) \in \mathbb{R}^2, \\ f_{yy}(x,y) = xe^y \qquad \forall (x,y) \in \mathbb{R}^2. \end{gather*}

    La matrice hessiana di f in (1,1) soddisfa

    (69)   \begin{equation*} \nabla^2 f(1,1) = \begin{pmatrix} -e		&	0 \\ 0	&	e, \end{pmatrix} \quad \Longrightarrow \quad \det \nabla^2 f(1,1) = -e^2<0, \end{equation*}

    quindi \nabla^2 f(1,1) è indefinita per la proposizione 1.13 e il punto (1,1) è di sella per il teorema 1.10.


 

Esercizio 4  (\bigstar\bigstar\largewhitestar\largewhitestar\largewhitestar). Determinare estremi assoluti e locali della funzione f \colon \mathbb{R}^2 \to \mathbb{R} definita da

(70)   \begin{equation*} f(x,y) = \sin x + \sin y + \sin(x+y) \qquad \forall (x,y) \in \mathbb{R}^2. \end{equation*}

Svolgimento.

La funzione f è di classe \mathcal{C}^{\infty}(\mathbb{R}^2) poiché somma di funzioni \sin ed è periodica di periodo 2\pi sia rispetto alla variabile x che rispetto alla variabile y, ossia

(71)   \begin{equation*} f(x+2k\pi,y) = f(x,y) = f(x,y+2k \pi) \qquad \forall (x,y) \in \mathbb{R}^2, \,\,\, \forall k \in \mathbb{Z}. \end{equation*}

In virtù di tali periodicità, è sufficiente studiare f nel quadrato

(72)   \begin{equation*} D \coloneqq [0,2\pi] \times [0,2\pi]. \end{equation*}

Poiché D è compatto e f è continua, per il teorema di Weierstrass 1.8 essa assume massimo e minimo assoluto in D. Inoltre, per la periodicità di f, questi coincidono con il massimo e il minimo assoluti di f in \mathbb{R}^2, in formule:

(73)   \begin{equation*} \max_D f = \max_{\mathbb{R}^2} f, \qquad \min_D f = \min_{\mathbb{R}^2} f. \end{equation*}

In particolare f possiede minimo e massimo assoluti su \mathbb{R}^2. Dato che essa è di classe \mathcal{C}^{\infty}(\mathbb{R}^2), per il teorema di Fermat 1.9 i punti di massimo e minimo assoluti e relativi vanno ricercati tra i punti stazionari di f in \mathbb{R}^2. Inoltre, per la periodicità di f notata precedentemente, è sufficiente studiare i soli punti stazionari appartenenti a D. Calcoliamo quindi le derivate parziali prime di f:

(74)   \begin{equation*} \begin{gathered} f_x(x,y) = \cos x + \cos(x+y) \qquad \forall (x,y) \in D, \\ f_y(x,y) = \cos y + \cos(x+y) \qquad \forall (x,y) \in D. \end{gathered} \end{equation*}

Si ha

(75)   \begin{equation*} \begin{split} \nabla f(x,y) = (0,0) \iff & \begin{cases} \cos x + \cos(x+y) = 0\\ \cos y + \cos(x+y) = 0 \end{cases} \\ \iff & \begin{cases} \cos x = \cos y\\ \cos y + \cos(x+y) = 0 \end{cases} \\ \iff & \begin{cases} x 	=	y\\[6pt] \cos y + \cos(2y) = 0 \end{cases} \vee \begin{cases} x=2\pi- y\\[6pt] \cos y + 1 = 0. \end{cases} \end{split} \end{equation*}

Per il primo sistema, ricordando la formula di duplicazione del coseno

(76)   \begin{equation*} \cos (2y)= \cos^2 y - \sin^2 y = 2 \cos^2y  -1 \qquad \forall y \in \mathbb{R}, \end{equation*}

si ottiene che è equivalente a

(77)   \begin{equation*} \begin{cases} x 	=	y\\[6pt] \cos y + 2 \cos^2 y - 1 = 0 \end{cases} \iff \begin{cases} x 	=	y\\[6pt] \cos y  = \dfrac{-1 \pm \sqrt{1+8}}{4}, \end{cases} \iff \begin{cases} x 	=	y\\[6pt] \cos y  = -1 \vee \cos y = \dfrac{1}{2}, \end{cases} \end{equation*}

che quindi possiede le soluzioni in D

(78)   \begin{equation*} (x,y) \in  \left \{ \left(  \pi, \pi \right), \left(  \frac{\pi}{3}, \frac{\pi}{3} \right), \left(  \frac{5\pi}{3}, \frac{5\pi}{3} \right) \right \}. \end{equation*}

Il secondo sistema in (75) possiede invece l’unica soluzione in D

(79)   \begin{equation*} (x,y)  = \left(  \pi, \pi \right), \end{equation*}

che era già una soluzione del sistema analizzato prima. Pertanto i punti stazionari di f in D sono tutti e soli quelli nell’insieme in (78). Per determinarne la natura, calcoliamo le derivate seconde di f al fine di applicare il teorema 1.10:

(80)   \begin{equation*} \begin{gathered} f_{xx}(x,y) = -\sin x - \sin(x+y) \qquad \forall (x,y) \in D, \\[4pt] f_{xy}(x,y) = f_{yx}(x,y) = - \sin(x+y) \qquad \forall (x,y) \in D, \\[4pt] f_{yy}(x,y) = -\sin y - \sin(x+y) \qquad \forall (x,y) \in D. \end{gathered} \end{equation*}

\bullet (\pi,\pi). Vale

(81)   \begin{equation*} \nabla^2 f(\pi,\pi) = \begin{pmatrix} 0	&	0\\ 0	&	0 \end{pmatrix}, \end{equation*}

che quindi è nulla e il teorema 1.10 non fornisce alcuna conclusione. Per stabilire la natura del punto stazionario (\pi,\pi), consideriamo 2 metodi.

    \[\quad\]

  1. Osserviamo che restringendoci alla retta di equazione x=y, si ha

    (82)   \begin{equation*} f(x,x) = 2\sin x +\sin(2x) = 2\sin x (1+\cos x) \qquad \forall x \in \mathbb{R}, \end{equation*}

    dove abbiamo usato la formula di duplicazione del seno \sin (2x)=2\sin x \cos x. Poiché 1+\cos x \geq 0 ed è strettamente positivo per x \in \left (\dfrac{\pi}{2}, \dfrac{3}{2}\pi \right ), mentre \sin x>0 per x \in \left (\dfrac{\pi}{2}, \pi\right ) e \sin x<0 per x \in \left (\pi, \dfrac{3}{2}\pi\right ), abbiamo

    (83)   \begin{equation*} f(x,x) >0  \quad \text{se } x \in \left (\frac{\pi}{2}, \pi\right ), \qquad f(x,x) <0  \quad \text{se } x \in \left (\pi, \frac{3}{2}\pi\right ), \end{equation*}

    ossia il punto (\pi,\pi) è di sella per f in quanto in ogni suo intorno f assume valori di segno opposto.

  2.  

  3. Partendo dal punto (\pi,\pi) e spostandosi lungo la retta di equazione y=\pi, il valore della funzione rimane invariato.

    (84)   \begin{equation*} f(\pi+h,\pi) = \sin(\pi+h) + \sin \pi + \sin (\pi + h + \pi) = -\sin h + \sin h = 0 = f(\pi,\pi) \qquad \forall h \in \mathbb{R}. \end{equation*}

    Fissando h \in \left ( 0,\dfrac{\pi}{2} \right ), si ha

    (85)   \begin{equation*} f_y(\pi+h,y) = \cos y + \cos(\pi+h+\pi) = \cos y  + \cos h < 0 \qquad \forall y \in (\pi-h,\pi+h), \end{equation*}

    come mostrato in figura 4, in cui rappresentiamo il fatto che la restrizione di f alla retta di equazione x=\pi+h sia strettamente decrescente nell’intervallo (\pi-h,\pi + h). Dunque

    (86)   \begin{equation*} f(\pi+h,y)<0 \quad \forall y \in (\pi,\pi+h), \qquad f(\pi+h,y)>0 \quad \forall y \in (\pi-h,\pi). \end{equation*}

    Per l’arbitrarietà di h \in \left ( 0,\dfrac{\pi}{2} \right ), ciò mostra che f assume, in ogni intorno di (\pi,\pi), sia valori maggiori che minori di f(\pi,\pi) ed esso è dunque un punto di sella per f.

    \[\quad\]

    \[\quad\]

Figura 4: rappresentazione delle restrizioni di f alle rette di equazione y=\pi e x=\pi+h. Le frecce indicano la direzione in cui f è crescente ottenuta studiando rispettivamente il segno di f_y. Da (85) si ottiene che f è decrescente sulla retta x=\pi+h. Ciò mostra che f assume valori di segno opposto in ogni intorno di (\pi,\pi) e quindi che il punto (\pi,\pi) è di sella per f.

    \[\quad\]

    \[\quad\]

\bullet \left(  \dfrac{\pi}{3}, \dfrac{\pi}{3} \right). Abbiamo

(87)   \begin{equation*} \nabla^2 f\left(  \frac{\pi}{3}, \frac{\pi}{3} \right) = \begin{pmatrix} -\sqrt{3}					&	- \dfrac{\sqrt{3}}{2}	\\ - \dfrac{\sqrt{3}}{2}		& 	- \sqrt{3} \end{pmatrix} \quad \Longrightarrow \quad \det \nabla^2 f\left( \frac{\pi}{3}, \frac{\pi}{3} \right) 3- \frac{3}{4} = \frac{9}{4}>0, \end{equation*}

quindi la matrice hessiana è definita in quanto i suoi due autovalori hanno stesso segno. Poiché

(88)   \begin{equation*} \operatorname{Tr} \nabla^2 f\left(  \frac{\pi}{3}, \frac{\pi}{3} \right) = f_{xx} \left(  \frac{\pi}{3}, \frac{\pi}{3} \right) + f_{yy}\left(  \frac{\pi}{3}, \frac{\pi}{3} \right) = -2\sqrt{3} < 0, \end{equation*}

per la proposizione 1.13 \nabla^2 f\left(  \dfrac{\pi}{3}, \dfrac{\pi}{3} \right) è definita negativa e dunque \left(  \dfrac{\pi}{3}, \dfrac{\pi}{3} \right) è un punto di massimo relativo per f per il teorema 1.10.

\bullet \left(  \dfrac{5\pi}{3}, \dfrac{5\pi}{3} \right). Poiché

(89)   \begin{equation*} f(x,y) = - f \left ( 2\pi - x, 2\pi-y \right ) \qquad \forall x,y \in D, \end{equation*}

e poiché \left(  \dfrac{5\pi}{3}, \dfrac{5\pi}{3} \right) =\left(  2\pi - \dfrac{\pi}{3}, 2\pi - \dfrac{\pi}{3} \right), per il punto precedente si ha che \left(  \dfrac{5\pi}{3}, \dfrac{5\pi}{3} \right) è di minimo locale per f.

Dato che i punti di estremo assoluto vanno ricercati tra quelli di estremo locale per f, si ha che

(90)   \begin{equation*} \left(  \frac{\pi}{3} + 2k\pi, \frac{\pi}{3} +2h\pi\right), \qquad \left(  -\frac{\pi}{3} + 2k\pi, -\frac{\pi}{3}+2h\pi\right) \qquad \forall h,k \in \mathbb{Z} \end{equation*}

sono rispettivamente punti di massimo e di minimo assoluti per f. Calcolando f in tali punti si ottiene quindi

(91)   \begin{equation*} \max_{\mathbb{R}^2} f = f\left(  \frac{\pi}{3}, \frac{\pi}{3}\right) =\frac{3 \sqrt{3}}{2}, \qquad \min_{\mathbb{R}^2} f = f\left(  \frac{5\pi}{3}, \frac{5\pi}{3} \right) =-\frac{3 \sqrt{3}}{2}. \end{equation*}


 

Esercizio 5  (\bigstar\bigstar\largewhitestar\largewhitestar\largewhitestar). Determinare estremi assoluti e locali della funzione f \colon \mathbb{R}^2 \to \mathbb{R} definita da

(92)   \begin{equation*} f(x,y) = (x-y)^4 - 8(x-y)^2 \qquad \forall (x,y) \in \mathbb{R}^2. \end{equation*}

Svolgimento.

Osserviamo che f non dipende dai particolari valori di x e y, ma solo dal valore della differenza x-y, ossia f è costante sulle rette di equazione x-y=t, al variare di t \in \mathbb{R}. Pertanto, ponendo appunto t \coloneqq x-y, studiare f si riduce a studiare la funzione f \colon \mathbb{R} \to \mathbb{R} definita da

(93)   \begin{equation*} g(t) = t^4 - 8 t^2 \qquad \forall t \in \mathbb{R}. \end{equation*}

Osserviamo innanzitutto che g è una funzione pari, quindi possiamo limitarci a studiarla nell’intervallo [0,+\infty). Poiché \lim_{t \to +\infty} g(t)=+\infty, si ha

(94)   \begin{equation*} +\infty = \sup_{t \in \mathbb{R}} g(t) = \sup_{(x,y) \in \mathbb{R}^2} f(x,y). \end{equation*}

Affermiamo poi che g possiede minimo assoluto. Infatti, notiamo innanzitutto che, essendo un polinomio, g è di classe \mathcal{C}^{\infty}(\mathbb{R}). Poiché \lim_{t \to +\infty} g(t)=+\infty, esiste T>0 tale che

(95)   \begin{equation*} g(t)>0 \qquad \forall t \geq T. \end{equation*}

Pertanto, applicando il teorema di Weierstrass 1.8 all’intervallo [0,T], g possiede minimo assoluto su tale intervallo e questo minimo deve coincidere col minimo assoluto di g su \mathbb{R} a causa di (95) e da g(0)=0. Quindi

(96)   \begin{equation*} \min_{\mathbb{R}} g = \min_{\mathbb{R}^2} f \leq 0. \end{equation*}

Per determinare i punti di minimo e per studiare altri eventuali estremi relativi di g (e quindi di f), studiamo g':

(97)   \begin{equation*} g'(t) = 4t^3-16t \qquad \forall t \in \mathbb{R}. \end{equation*}

Limitandoci all’intervallo [0,+\infty), abbiamo

(98)   \begin{equation*} g'(t) = 4t(t^2-4) \quad \begin{cases} > 0 			& \text{se } t > 2\\ \leq 0			& \text{se } t \in [0,2], \end{cases} \end{equation*}

dunque da ciò si ha che t=0 è un punto di massimo relativo per g, mentre t=\pm 2 sono dei punti di minimo relativo per g. Poiché esiste il minimo assoluto di g, i punti t=\pm 2 sono anche di minimo assoluto per g.

Ritornando a f si ha che la retta di equazione x-y=0 è un insieme di punti di massimo relativo per f, mentre le rette di equazione x-y=\pm 2 sono costituite da punti di minimo assoluto per f, dunque

    \begin{equation*} \min_{\mathbb{R}^2} f = 2^4 - 8 \cdot 2^2 = -16. \end{equation*}


Svolgimento alternativo.

Vogliamo presentare una seconda soluzione dell’esercizio, che non fa uso della semplificazione data dal ricondurre f a una funzione di una sola variabile.

Osserviamo innanzitutto che f \in \mathcal{C}^{\infty}(\mathbb{R}^2) poiché è un polinomio e che

(99)   \begin{equation*} \lim_{x \to + \infty} f(x,-x) = \lim_{x \to + \infty} (2x)^4 - 8(2x)^2 = +\infty, \end{equation*}

dunque

(100)   \begin{equation*} \sup_{\mathbb{R}^2} f= + \infty. \end{equation*}

Per il minimo, abbiamo invece che

(101)   \begin{equation*} f(x,y) = (x-y)^4 - 8(x-y) + 16 - 16 = \big( (x-y)^2 - 4 \big)^2 - 16 \geq -16 \qquad \forall (x,y) \in \mathbb{R}^2. \end{equation*}

Dato che l’uguaglianza vale se e solo se (x-y)^2 = 4, ovvero se e solo se x-y= \pm 2, si ha che queste rette sono di minimo assoluto per f e

(102)   \begin{equation*} \min_{\mathbb{R}^2} f = -16. \end{equation*}

Per ricercare altri punti di estremo relativo per f, utilizzando il teorema 1.9 e conducendo lo studio classico dei punti critici di f, si ottiene che essi sono costituiti dalle rette

(103)   \begin{equation*} x-y=-2, \qquad x-y=0, \qquad x-y=2, \end{equation*}

e che ogni punto di tali rette possiede determinante hessiano nullo. Ciò può essere ottenuto dal calcolo diretto, ma vogliamo qui presentare un altro argomento. Poiché la funzione è costante su tutte le rette parallele al vettore (1,1), questo vettore deve necessariamente annullare la forma quadratica associata alla matrice hessiana.

Per mostrare questa affermazione rigorosamente, fissiamo c \in \mathbb{R} e restringiamo la funzione f a una retta di equazione y=x+c, ottenendo la funzione g_c \colon \mathbb{R} \to \mathbb{R} definita da

(104)   \begin{equation*} g_c(x)=f(x,x-c)= c^4 - 8c^2 \qquad \forall x \in \mathbb{R}, \end{equation*}

che è costante. Ciò implica che tutte le sue derivate sono nulle e dunque, applicando 2 volte la regola della catena [5, teorema 3] alla funzione composta

(105)   \begin{equation*} g_c \colon x \in \mathbb{R} \mapsto (x,x-c) \in \mathbb{R}^2 \mapsto f(x,x-c) \in \mathbb{R}, \end{equation*}

si ottiene

(106)   \begin{gather*} 0 = g'_c(x) = f_x(x,x-c) + f_y(x,x-c) = \nabla f(x,x-c) \begin{pmatrix} 1\\ 1 \end{pmatrix} \qquad \forall x \in \mathbb{R}, \\[4pt] 0 = g''_c(x) = f_{xx}(x,x-c) + 2f_{xy}(x,x-c) + f_y(x,x-c) = \begin{pmatrix} 1	& 1 \end{pmatrix} \nabla^2 f(x,x-c) \begin{pmatrix} 1\\ 1 \end{pmatrix} \qquad \forall x \in \mathbb{R}. \end{gather*}

Dalla seconda equazione si evince che (1,1) appartiene al nucleo di \nabla^2 f, che risulta quindi semidefinita.

Per studiare la natura di tali punti critici, si sarebbero potute effettuare le seguenti osservazioni.

\bullet retta x-y=0. Si ha

(107)   \begin{equation*} f(x,y) = (x-y)^2 \left ( (x-y)^2-8 \right ) \qquad \forall (x,y) \in \mathbb{R}. \end{equation*}

Mentre il fattore (x-y)^2 è sempre positivo e si annulla solo per x-y=0, il fattore (x-y)^2-8 è negativo se |x-y|< \sqrt{8}, ossia in un intorno dei punti della retta x-y=0. Pertanto si ha

(108)   \begin{equation*} f(x,y) < 0 \qquad \forall (x,y) \in \mathbb{R}^2 \colon 0< |x-y| < \sqrt{8}, \end{equation*}

da cui segue che la retta x-y=0, su cui f vale 0, è costituita da punti di massimo relativo per f.

\bullet rette x-y=\pm 2. Abbiamo già osservato che i punti delle rette di equazione x - y = \pm 2 sono di minimo assoluto per f.


 

Esercizio 6  (\bigstar\bigstar\bigstar\largewhitestar\largewhitestar). Determinare estremi assoluti e locali della funzione f \colon \mathbb{R}^2 \to \mathbb{R} definita da

(109)   \begin{equation*} f(x,y) = x^4+y^4 -2(x-y)^2 \qquad \forall (x,y) \in \mathbb{R}^2. \end{equation*}

Svolgimento.

Osserviamo innanzitutto che, restringendoci alla retta di equazione x=y, si ha

(110)   \begin{equation*} \lim_{x \to \pm \infty} f(x,x) = \lim_{x \to \pm \infty} 2x^4 = +\infty, \end{equation*}

da cui segue che

(111)   \begin{equation*} \sup_{\mathbb{R}^2} f = +\infty. \end{equation*}

Affermiamo invece che f possiede minimo assoluto su \mathbb{R}^2. Infatti cominciamo con l’osservare che

(112)   \begin{equation*} (x-y)^2 = x^2 -2xy + y^2 \leq 2x^2 + 2y^2 \qquad \forall (x,y) \in \mathbb{R}^2, \end{equation*}

dove nella disuguaglianza si è usato che x^2 +2xy + y^2 = (x+y)^2 \geq 0 e quindi -2xy \leq x^2+y^2. Inserendo (112) nell’espressione di f, si ottiene

(113)   \begin{equation*} f(x,y) \geq x^4+y^4 - 2x^2 - 2y^2 = x^2(x^2-2) + y^2(y^2-2) > 0 \qquad \forall (x,y) \in \mathbb{R}^2 \colon |x|>\sqrt{2}, |y|> \sqrt{2}. \end{equation*}

Poiché f è polinomiale, essa è di classe \mathcal{C}^{\infty}(\mathbb{R}^2) e dunque si può applicare il teorema di Weierstrass 1.8 a essa sul quadrato

(114)   \begin{equation*} Q = \{(x,y) \in \mathbb{R}^2 \colon |x|\leq\sqrt{2}, |y|\leq \sqrt{2}\}, \end{equation*}

che è compatto, e quindi stabilire che f possiede minimo assoluto su Q. Poiché f(0,0)=0, si ha \min_Q f \leq 0 e da (113) segue che

(115)   \begin{equation*} \min_Q f = \min_{\mathbb{R}^2} f, \end{equation*}

che quindi esiste. Poiché tale minimo assoluto è anche relativo, per determinarne il valore usiamo il teorema di Fermat 1.9 e studiamo i punti critici di f.

(116)   \begin{equation*} \begin{gathered} f_x(x,y) = 4x^3 - 4(x-y) \qquad \forall (x,y) \in \mathbb{R}^2, \\ f_y(x,y) = 4y^3 + 4(x-y) \qquad \forall (x,y) \in \mathbb{R}^2. \end{gathered} \end{equation*}

Si ha

(117)   \begin{equation*} \begin{split} \nabla f(x,y) = (0,0) \iff & \begin{cases} 4x^3 = 4(x-y)\\ 4y^3 + 4(x-y) = 0 \end{cases} \\ \iff & \begin{cases} x^3 = x-y\\ y^3 + x^3 = 0 \end{cases} \\ \iff & \begin{cases} x^3 	=	2x\\ y = -x. \end{cases} \end{split} \end{equation*}

Scrivendo x^3-2x = x(x^2-2) = x(x-\sqrt{2})(x+\sqrt{2}), si vede che le soluzioni di tale sistema sono

(118)   \begin{equation*} (0,0), \qquad (\sqrt{2},-\sqrt{2}), \qquad (-\sqrt{2},\sqrt{2}). \end{equation*}

Osservando che

(119)   \begin{equation*} f(-x,-y)=f(x,y) \qquad \forall (x,y) \in \mathbb{R}^2, \end{equation*}

possiamo limitarci a studiare soltanto i punti (0,0) e (\sqrt{2},-\sqrt{2}). A tal fine proviamo a usare il criterio fornito dal teorema 1.10 e calcoliamo le derivate seconde di f.

(120)   \begin{equation*} \begin{gathered} f_{xx}(x,y) = 12x^2 - 4 \qquad \forall (x,y) \in \mathbb{R}^2, \\ f_{xy}(x,y) = f_{yx}(x,y) = 4 \qquad \forall (x,y) \in \mathbb{R}^2, \\ f_{yy}(x,y) = 12y^2 - 4 \qquad \forall (x,y) \in \mathbb{R}^2. \end{gathered} \end{equation*}

\bullet (0,0). Si ha

(121)   \begin{equation*} \nabla^2 f(0,0) = \begin{pmatrix} -4		&	4\\ 4		&	-4 \end{pmatrix} \quad \Longrightarrow \quad \det \nabla^2 f(0,0) = 0, \end{equation*}

da cui segue che \nabla^2 f(0,0) è semidefinita per la proposizione 1.13. Osservando che \operatorname{Tr} \nabla^2 f(0,0) < 0, di nuovo la proposizione 1.13 implica che \nabla^2 f(0,0) è semidefinita negativa; poiché essa non è nulla, per il teorema 1.10 il punto (0,0) è di sella o di massimo relativo per f. Osserviamo che, poiché per x=y il termine negativo nell’espressione di f si annulla, è naturale osservare che

(122)   \begin{equation*} f(x,x) = 2x^4 > 0 \qquad \forall x \neq 0. \end{equation*}

D’altra parte, poiché per x vicino a zero ci si aspetta che il termine quadratico sia maggiore in modulo del termine avente esponente 4, studiamo

(123)   \begin{equation*} f(x,0) = x^4 - 2x^2 = x^2(x^2-2) < 0 \qquad \forall x \in (-\sqrt{2},\sqrt{2}) \setminus \{0\}. \end{equation*}

Confrontando (122) e (123), si vede che f assume valori di segno opposto in ogni intorno di (0,0), dunque il punto (0,0) è di sella per f.

\bullet (\sqrt{2},-\sqrt{2}). Poiché abbiamo dimostrato che f possiede almeno un punto di minimo assoluto, l’unica possibilità per (\sqrt{2},-\sqrt{2}) è appunto questa. Dall’osservazione (119) possiamo quindi concludere che (\sqrt{2},-\sqrt{2}) e (-\sqrt{2},\sqrt{2}) sono punti di minimo assoluto per f e dunque

(124)   \begin{equation*} \min_{\mathbb{R}^2} f = f(\sqrt{2},-\sqrt{2}) = -8. \end{equation*}

Volendo invece studiare la natura di questi punti stazionari in maniera più diretta, si può osservare che

(125)   \begin{equation*} \nabla^2 f (\sqrt{2},-\sqrt{2}) = \begin{pmatrix} 20		&	4\\ 4		&	20 \end{pmatrix} \quad \Longrightarrow \quad \det \nabla^2 f(\sqrt{2},-\sqrt{2}) > 0, \quad \operatorname{Tr} f(\sqrt{2},-\sqrt{2}) > 0 \end{equation*}

e quindi \nabla^2 f(\sqrt{2},-\sqrt{2}) è definita positiva per la proposizione 1.13. Per il teorema 1.10, (\sqrt{2},-\sqrt{2}) è un punto di minimo locale per f, che è anche assoluto in quanto esso è, a meno della parità di f in (119), l’unico punto di estremo locale e poiché abbiamo stabilito in precedenza l’esistenza di punti di minimo assoluto per f.


 

Esercizio 7  (\bigstar\bigstar\bigstar\largewhitestar\largewhitestar). Determinare estremi assoluti e locali della funzione f \colon \mathbb{R}^2 \to \mathbb{R} definita da

(126)   \begin{equation*} f(x,y) = (y-3x^2)(y-x^2) \qquad \forall (x,y) \in \mathbb{R}^2. \end{equation*}

Svolgimento.

Osserviamo che la funzione f presenta delle affinità di forma con le parabole di equazione y=cx^2 con c \in \mathbb{R}. Risulta naturale quindi studiare f su tali parabole. Osserviamo che

(127)   \begin{equation*} f(x,cx^2) >0 \iff c>3 \vee c<1, \qquad f(x,cx^2) < 0 \iff c \in (1,3), \end{equation*}

situazione rappresentata in figura 5.

    \[\quad\]

    \[\quad\]

Figura 5: {segni assunti dalla funzione f. In rosso le aree in cui f assume segno positivo, in blu le aree dove f assume segno negativo. Dall’espressione di f vede che essa assume segno positivo per y>3x^2 e per y < x^2, mentre assume segno negativo nella regione compresa tra le due parabole, in cui i fattori assumono segno opposto.

    \[\quad\]

    \[\quad\]

Considerando tale informazione, osserviamo che

(128)   \begin{equation*} \lim_{x \to +\infty} f(x,2x^2) = \lim_{x \to + \infty} -x^4 = - \infty, \qquad \lim_{x \to +\infty} f(x,0) = \lim_{x \to +\infty} 3x^4 = +\infty, \end{equation*}

da cui segue che

(129)   \begin{equation*} \inf_{\mathbb{R}^2} f = - \infty, \qquad \sup_{\mathbb{R}^2} f = + \infty. \end{equation*}

Per determinare eventuali estremi relativi di f, poiché essa è di classe \mathcal{C}^{\infty}(\mathbb{R}^2), usiamo il teorema di Fermat 1.9, ricercandoli tra i suoi punti stazionari. Quindi calcoliamo le derivate parziali di f. A tal fine, svolgiamo i prodotti nell’espressione di f ottenendo

(130)   \begin{equation*} f(x,y) = y^2 - 4x^2y + 3 x^4 \qquad \forall (x,y) \in \mathbb{R}^2. \end{equation*}

Si ha

(131)   \begin{equation*} \begin{gathered} f_x(x,y) = -8xy + 12 x^3 \qquad \forall (x,y) \in \mathbb{R}^2, \\ f_y(x,y) = 2y - 4x^2 \qquad \forall (x,y) \in \mathbb{R}^2. \end{gathered} \end{equation*}

Ricercando i punti stazionari di f si ottiene

(132)   \begin{equation*} \begin{split} \nabla f(x,y) = (0,0) \iff & \begin{cases} -2xy + 3 x^3=0\\ y - 2x^2 = 0 \end{cases} \\ \iff & \begin{cases} x = 0 \,\, \vee \,\, y= \dfrac{3}{2}x^2 \\[6pt] y = 2x^2, \end{cases} \end{split} \end{equation*}

che come unica soluzione possiede (x,y)=(0,0).

Per studiare la natura di tale punto critico, proviamo a utilizzare il teorema 1.10 e osserviamo che

(133)   \begin{equation*} \begin{gathered} f_{xx}(x,y) = -8y + 36x^2 \qquad \forall (x,y) \in \mathbb{R}^2, \\ f_{xy}(x,y) = f_{yx}(x,y) = -8x \qquad \forall (x,y) \in \mathbb{R}^2, \\ f_{yy}(x,y) = 2 \qquad \forall (x,y) \in \mathbb{R}^2. \end{gathered} \end{equation*}

La matrice hessiana di f in (0,0) è pari a

(134)   \begin{equation*} \nabla^2 f(0,0) = \begin{pmatrix} 0		&	0\\ 0		&	2 \end{pmatrix}, \end{equation*}

che risulta quindi semidefinita positiva, avendo un autovalore nullo e uno pari a 2. Osserviamo però da (127) e dalla figura 5 che f assume segni di valore opposto in ogni intorno di (0,0), pertanto tale punto è di sella per f.


 

Esercizio 8  (\bigstar\bigstar\largewhitestar\largewhitestar\largewhitestar). Determinare estremi assoluti e locali della funzione f \colon \mathbb{R}^2 \to \mathbb{R} definita da

(135)   \begin{equation*} f(x,y) = 4 - \sqrt{x^2+4y^2} \qquad \forall (x,y) \in \mathbb{R}^2. \end{equation*}

Svolgimento.

Poiché la quantità \sqrt{x^2+4y^2} è non-negativa, si vede subito che

(136)   \begin{equation*} f(x,y) \leq 4 \qquad \forall (x,y) \in \mathbb{R}^2, \end{equation*}

e che tale valore è assunto se e solo se (x,y)=(0,0). Quindi tale punto è di massimo assoluto per f e

(137)   \begin{equation*} \max_{\mathbb{R}^2} f = 4. \end{equation*}

Per studiare gli altri estremi della funzione, osserviamo innanzitutto che i valori assunti da f dipendono soltanto dalla quantità x^2+4y^2, quindi essa è costante sulle curve di equazione

(138)   \begin{equation*} \sqrt{x^2+4y^2} = t \qquad \text{con } t \in [0,+\infty), \end{equation*}

che sono delle ellissi aventi centro nell’origine degli assi e che, al variare di t \in [0,+\infty), ricopriono l’intero piano cartesiano \mathbb{R}^2. Quindi lo studio della funzione f si riconduce a quello della funzione g \colon [0,+\infty) \to \mathbb{R} definita da

(139)   \begin{equation*} g(t)= 4 - t \qquad \forall t \in [0,+\infty). \end{equation*}

Si vede subito che g è strettamente decrescente in [0,+\infty), quindi l’unico punto di estremo relativo per essa è lo 0, che è un massimo assoluto e corrisponde al punto (0,0) di massimo assoluto per f. Dunque nemmeno f ha altri punti di estremo relativo. Si ha poi

(140)   \begin{equation*} \lim_{t \to + \infty} g(t) = \lim_{t \to + \infty} 4- t = -\infty, \end{equation*}

da cui segue che

(141)   \begin{equation*} \inf_{t \in \mathbb{R}} g(t) = \inf_{(x,y) \in \mathbb{R}^2} f(x,y) = - \infty. \end{equation*}

Osserviamo che, se avessimo tentato di studiare gli estremi di f col metodo classico, avremmo trovato che essa non possiede derivate parziali in (0,0) e che non possiede punti critici.


 

Esercizio 9  (\bigstar\bigstar\bigstar\largewhitestar\largewhitestar). Sia a \in \mathbb{R} e si consideri la funzione f_a \colon \mathbb{R}^2 \to \mathbb{R} definita da

(142)   \begin{equation*} f_a(x,y) = \log(1+x^2+y^2) + axy \qquad \forall (x,y) \in \mathbb{R}^2. \end{equation*}

Dopo aver mostrato che, per ogni a \in \mathbb{R}, il punto (0,0) è critico per f_a, determinarne la natura al variare di a \in \mathbb{R}.

Svolgimento.

f è di classe \mathcal{C}^{\infty}(\mathbb{R}^2) in quanto somma e composizione di polinomi e logaritmi. Calcoliamo le derivate parziali di f_a:

(143)   \begin{equation*} \begin{gathered} \frac{\partial f_a}{\partial x}(x,y) = \frac{2x}{1+x^2+y^2} + ay \qquad \forall (x,y) \in \mathbb{R}^2, \\[7pt] \frac{\partial f_a}{\partial y}(x,y) = \frac{2y}{1+x^2+y^2} + ax \qquad \forall (x,y) \in \mathbb{R}^2. \end{gathered} \end{equation*}

Si ha chiaramente

(144)   \begin{equation*} \nabla f_a(0,0) = \left ( \frac{\partial f_a}{\partial x}(0,0), \frac{\partial f_a}{\partial y}(0,0) \right ) = (0,0) \qquad \forall a \in \mathbb{R}, \end{equation*}

dunque (0,0) è un punto stazionario per ognuna delle f_a. Per stabilirne la natura, possiamo utilizzare il teorema 1.10 e calcolare la matrice hessiana di f_a in (0,0). Invece di calcolare le derivate seconde col metodo classico, offriamo al lettore un metodo alternativo che può risultare utile quando le derivate sono complicate da calcolare analiticamente e che fa uso dei polinomi di Taylor.

Osserviamo che, dal fatto noto che \log (1+t) = t - \dfrac{t^2}{2} +o(t^2) per t \to 0, e poiché occorre studiare f_a in un intorno di (0,0), abbiamo che

(145)   \begin{equation*} f_a(x,y) = x^2+y^2 - \frac{1}{2}(x^2+y^2)^2 + o(x^2+y^2)^2 + axy \qquad \forall (x,y) \in \mathbb{R}^2. \end{equation*}

Da tale espressione segue che il polinomio di Taylor P_a(x,y) di f di ordine 2 centrato in (0,0) è proprio pari a

(146)   \begin{equation*} P_a(x,y) = x^2+y^2+a xy \end{equation*}

e pertanto da esso, grazie al teorema di Taylor [3, sezione 35], si possono ricavare le derivate seconde di f in (0,0):

(147)   \begin{equation*} \begin{gathered} \frac{\partial^2 f_a}{\partial x^2}(0,0) = \frac{\partial^2 P_a}{\partial x^2}(0,0) = 2, \\ \frac{\partial^2 f_a}{\partial xy}(0,0) = \frac{\partial^2 f_a}{\partial yx}(0,0) = \frac{\partial^2 P_a}{\partial xy}(0,0) = a, \\ \frac{\partial^2 f_a}{\partial y^2}(0,0) = \frac{\partial^2 P_a}{\partial y^2}(0,0) = 2, \end{gathered} \end{equation*}

da cui quindi segue che la matrice hessiana di f_a in (0,0) soddisfa

(148)   \begin{equation*} \nabla^2 f_a(0,0) = \begin{pmatrix} 2		& 	a\\ a		&	2 \end{pmatrix} \quad \Longrightarrow \quad \det \nabla^2 f_a(0,0) = 4 - a^2 \quad \begin{cases} >0			& \text{se } a \in (-2,2)\\ =0			& \text{se } a \in \{-2,2\}\\ <0			& \text{se } a \in (-\infty,-2) \cup (2,+\infty). \end{cases} \end{equation*}

Da tale casistica deduciamo la seguente classificazione.

    \[\quad\]

  • Se a \in (-2,2), il determinante hessiano è positivo e quindi \nabla^2 f_a(0,0) possiede autovalori dello stesso segno. Tali autovalori sono positivi in quanto \operatorname{Tr} \nabla^2 f_a(0,0)=4>0, ovvero la loro somma è positiva. Dunque \nabla^2 f_a(0,0) è definita positiva e quindi (0,0) è un punto di minimo rel
  •  

  • Se a \in (-\infty,-2) \cup (2,+\infty), il determinante hessiano è negativo e quindi la matrice hessiana è indefinita per la proposizione 1.13. Pertanto il punto (0,0) è di sella per f per il teorema 1.10.
  •  

  • Se a\in \{-2,2\}, il determinante hessiano è nullo e la matrice hessiana \nabla^2 f_a(0,0) possiede almeno un autovalore nullo. L’altro autovalore è positivo poiché \operatorname{Tr} \nabla^2 f_a(0,0)=4>0, dunque \nabla^2 f_a(0,0) è semidefinita positiva. Dunque, per il teorema 1.10, il punto (0,0) può essere di minimo relativo o di sella. Per stabilirne la natura, fissiamo innanzitutto a=2, in quanto il caso a=-2 è analogo.

    Osserviamo che, ad esempio, sulla rette di equazione y=0 e y=-x si ha rispettivamente

    (149)   \begin{gather*} f_2(x,0) = \log(1+x^2)>0 \qquad \forall x \in \mathbb{R}, \\ f_2(x,-x) = \log(1+2x^2) -2x^2 = 2x^2 - \frac{1}{2} (2x^2)^2 + o(x^4) - 2x^2 = - 2x^4 + o(x^4) \qquad \forall x \in \mathbb{R}, \end{gather*}

    dove la seconda uguaglianza segue dallo sviluppo di Taylor

    (150)   \begin{equation*} \log(1+t) = t - \frac{1}{2}t^2 + o(t^2). \end{equation*}

    Si ha dunque

    (151)   \begin{equation*} f_2(x,-x) = x^4 \left ( - 2 + o(1) \right ) < 0 \quad \text{per } x \to 0. \end{equation*}

    Alternativamente, si poteva studiare la funzione g \colon \mathbb{R}\to \mathbb{R} definita da

    (152)   \begin{equation*} g(x) = f_2(x,-x) = \log(1+2x^2) - 2x^2 \qquad \forall x \in \mathbb{R}, \end{equation*}

    osservare che g è pari e che

    (153)   \begin{equation*} g'(x) = \frac{4x}{1+2x^2} - 4x < 0 \qquad \forall x \in (0,+\infty). \end{equation*}

    Pertanto g è strettamente decrescente in (0,+\infty) e, da g(0)=0, segue che

    (154)   \begin{equation*} g(x)<0 \qquad \forall x \in \mathbb{R} \setminus \{0\}. \end{equation*}

    Dunque f_2(x,-x)<0 per ogni x \in \mathbb{R} \setminus \{0\}. Da ciò e da (149) si ottiene che (0,0) è un punto di sella per f_2.


 

Esercizio 10  (\bigstar\bigstar\bigstar\bigstar\bigstar). Determinare estremi assoluti e locali della funzione f \colon \mathbb{R}^2 \to \mathbb{R} definita da

(155)   \begin{equation*} f(x,y) = \left (1 - \cos\left ( x^2-y\right ) \right )\sin(2x-y-1) \qquad \forall (x,y) \in \mathbb{R}^2. \end{equation*}

Svolgimento.

Applicare il metodo proposto negli esercizi precedenti alla funzione f può presentare notevoli difficoltà di calcolo: il lettore può provare a scrivere le espressioni delle derivate parziali di f e a risolvere il sistema \nabla f(x,y)= \mathbf{0} per rendersene conto. Per questo motivo proponiamo una strada alternativa che fa uso di un cambio di variabili.

Per chiarezza espositiva, descriviamo brevemente la strategia risolutiva che abbiamo adottato.

    \[\quad\]

  1. Definiamo il cambio di variabili (u,v)=(x^2-y,2x-y-1), osserviamo che (u,v) vivono nel semipiano D rappresentato in figura 6 e definiamo la funzione g ottenuta col cambio di variabili.
  2.  

  3. Studiamo la presenza di punti di estremo assoluto per g, sfruttando delle semplici disuguaglianze per le funzioni trigonometriche.
  4.  

  5. Determiniamo poi l’eventuale esistenza di punti di estremo relativo per g all’interno di D, usando il teorema di Fermat 1.9, il teorema 1.10 e altri metodi a causa della presenza di matrici hessiane semidefinite.
  6.  

  7. Mostriamo poi che g non possiede punti di estremo al bordo di D. In questo caso, non si può applicare il teorema di Fermat, che è invece relativo a punti interni al dominio. Occorre quindi utilizzare altre tecniche: cercheremo eventuali punti di estremo di g tra i punti di estremo della restrizione di g a \partial D, per poi verificare che essi sono dei punti di sella.
  8.  

  9. Infine, traduciamo per la funzione originaria f i risultati ottenuti per g.

Sviluppiamo nel seguito ognuno dei punti sopra descritti.

1. Cambio di variabili: definizione di g e D. La funzione f dipende solo dalle quantità x^2-y e 2x - y-1, pertanto un’idea è che sia conveniente definire le variabili

(156)   \begin{equation*} u \coloneqq {x^2-y}, \quad v \coloneqq 2x - y-1 \qquad \forall (x,y) \in \mathbb{R}^2 \end{equation*}

e studiare la funzione g \colon  \mathbb{R}^2 \to \mathbb{R} definita da

(157)   \begin{equation*} g(u,v) = \big( 1 - \cos u \big) \sin v \qquad \forall (u,v) \in \mathbb{R}^2. \end{equation*}

Si noti che la quantità u={x^2-y} rappresenta il termine noto della parabola con asse verticale di equazione y=x^2 - u, mentre la quantità v=2x-y-1 rappresenta il termine noto c della retta di equazione 2x -y -1 =v su cui si trova il punto (x,y).

È molto importante notare che le due variabili u=x^2-y e v=2x-y-1 non sono completamente indipendenti: formalmente, la funzione \Phi che definisce il cambio di variabili

(158)   \begin{equation*} \Phi \colon (x,y) \in \mathbb{R}^2 \longmapsto (u,v)=(x^2-y,2x-y-1) \in \mathbb{R}^2 \end{equation*}

non è suriettiva e ha immagine pari all’insieme

(159)   \begin{equation*} \operatorname{Im}\Phi = D \coloneqq \{ (u,v) \in \mathbb{R}^2 \colon  u  \in \mathbb{R},\,\, v \leq u \}. \end{equation*}

Per motivare questa affermazione, mostriamo che (u,v) \in \operatorname{Im}f se e solo se v \leq u. Infatti, (x,y) sono le coordinate di un punto del piano \mathbb{R}^2 se e solo se il sistema

(160)   \begin{equation*} \begin{cases} x^2 - y =  u \\ 2x - y - 1 = v \end{cases} \end{equation*}

ha soluzione. Sottraendo le equazioni, si vede che ciò implica che l’equazione x^2 - 2x + (1 - u + v)=0 ha soluzione, che è vero se e solo se 4 - 4(1 - u + v) \geq 0 ossia se e solo se u \geq v. D’altra parte, se u \geq v, invertendo il ragionamento si può ricavare

(161)   \begin{equation*} x= 1 \pm \sqrt{u-v}, \end{equation*}

che, sostituito in (160), produce le soluzioni

(162)   \begin{equation*} y=(1-v) \pm \sqrt{u-v}. \end{equation*}

Dunque il problema è equivalente a studiare g nell’insieme D, rappresentato in figura 6.

    \[\quad\]

    \[\quad\]

Figura 6: il dominio D, colorato in verde, in cui occorre studiare la funzione g.

    \[\quad\]

    \[\quad\]

2. Estremi assoluti di g. Notiamo innanzitutto che g (e quindi f) è limitata, in quanto

(163)   \begin{equation*} |g(u,v)| \leq |1-\cos u||\sin v| \leq 2 \qquad \forall (u,v) \in \mathbb{R}^2. \end{equation*}

Poiché g assume valore 2 se e solo se \cos u=-1 e \sin v=1, (163) implica che i punti di massimo assoluto per g sono tutti e soli i punti

(164)   \begin{equation*} (u,v) \in D \colon \cos u =-1 \,\, \wedge \,\, \sin v=1. \end{equation*}

In altre parole, l’insieme M_g dei punti di massimo assoluto per g è pari a

(165)   \begin{equation*} \begin{split} M_g = & \left \{ (u,v) \in D \colon u=\pi + 2h\pi, \,\, v= \frac{\pi}{2} + 2k\pi, \,\, h,k \in \right \} \\ = & \left \{ (u,v) \in \mathbb{R}^2 \colon u=\pi + 2h\pi, \,\, v= \frac{\pi}{2} + 2k\pi, \,\, h, k \in \mathbb{Z},\,\, k \leq h \right \}. \end{split} \end{equation*}

Analogamente, dato che g assume valore -2 se e solo se \cos u=-1 e \sin v=-1, da (163) segue che l’insieme dei punti m_g di minimo assoluto per g è costituito dai punti

(166)   \begin{equation*} (u,v) \in D \colon \cos u=-1 \,\, \wedge \,\, \sin v=-1, \end{equation*}

ossia

(167)   \begin{equation*} \begin{split} m_g = & \left \{ (u,v) \in D \colon u=\pi + 2h\pi, \,\, v= \frac{3}{2}\pi + 2k\pi, \,\, h,k \in \mathbb{Z} \right \} \\ = & \left \{ (u,v) \in D \colon u=\pi + 2h\pi, \,\, v= \frac{3}{2}\pi + 2k\pi, \,\, h, k \in \mathbb{Z},\,\, k \leq  h - 1 \right \}. \end{split} \end{equation*}

In particolare

(168)   \begin{equation*} \max_{D} g = \max_{\mathbb{R}^2} f = 2, \qquad \min_{D} g = \min_{\mathbb{R}^2} f = -2. \end{equation*}

Per determinare eventuali punti di estremo relativo per g, dobbiamo considerare separatamente la parte interna e il bordo del dominio D. Infatti, poiché il teorema 1.9 si applica solo ai punti interni al dominio della funzione, potrebbero esserci punti di estremo relativo sul bordo per cui le derivate parziali non si annullano e quindi tali punti vanno cercati con altri metodi che non utilizzano il teorema 1.9.

3. Estremi locali all’interno di D. Dato che g \in \mathcal{C}^{\infty}(D), per determinare i punti di estremo relativo interni a D usiamo il teorema di Fermat 1.9 e li ricerchiamo tra i suoi punti stazionari.

(169)   \begin{equation*} \begin{gathered} g_u(u,v) = \sin u \sin v \qquad \forall (u,v) \in \mathbb{R}^2, \\ g_v(u,v) = \big( 1 -\cos u \big)\cos v \qquad \forall (u,v) \in \mathbb{R}^2. \end{gathered} \end{equation*}

Il sistema che fornisce le coordinate dei punti stazionari di f è il seguente:

(170)   \begin{equation*} \begin{split} \nabla g(u,v) = (0,0) \iff & \begin{cases} \sin u = 0 \,\,\vee\,\, \sin v=0\\[4pt] 1-\cos u = 0 \,\,\vee \,\,\cos v=0 \end{cases} \\ \iff & \begin{cases} \sin u = 0\\[4pt] 1-\cos u = 0 \end{cases} \vee \,\, \begin{cases} \sin u = 0\\[4pt] \cos v = 0 \end{cases} \vee \,\, \begin{cases} \sin v = 0\\[4pt] 1-\cos u = 0 \end{cases} \vee \,\, \begin{cases} \sin v = 0\\[4pt] \cos v = 0. \end{cases} \end{split} \end{equation*}

Si vede subito che il quarto sistema è impossibile. Le soluzioni del primo sistema sono

(171)   \begin{equation*} \left \{ (u,v) \in D \colon u=2k\pi,\,\,k \in \mathbb{Z} \right \}. \end{equation*}

mentre le soluzioni del secondo e del terzo sono rispettivamente

(172)   \begin{equation*} \left \{ (u,v) \in D \colon u=k\pi,\,\, v= \frac{\pi}{2} + h\pi,\,\,h,k \in \mathbb{Z} \right \}, \qquad \left \{ (u,v) \in D \colon u=2k\pi,\,\, v= h\pi,\,\,h,k \in \mathbb{Z} \right \}. \end{equation*}

Dato che g è periodica di periodo 2\pi rispetto a u e a v ed è dispari rispetto a v, è sufficiente studiare i punti stazionari contenuti nell’intersezione del rettangolo [0,2\pi] \times [0,2\pi] con D. Da ciò, unificando le soluzioni dei vari sistemi, occorre quindi studiare la natura dei punti critici seguenti:

(173)   \begin{equation*} \begin{gathered} \left (\pi,\frac{\pi}{2}\right ), \qquad (2\pi,v) \quad \forall v \in [0,2\pi]. \end{gathered} \end{equation*}

Dalla discussione precedente sappiamo già che il punto \left (\pi,\frac{\pi}{2}\right ) è di massimo assoluto per f, dunque rimangono da studiare soltanto i punti

(174)   \begin{equation*} \begin{gathered} %(0,0), \qquad (2\pi,v) \quad \forall v \in [-2\pi,2\pi]. \end{gathered} \end{equation*}

Per determinarne la natura, si potrebbe tentare di usare il teorema 1.10, calcolando le derivate seconde di g:

(175)   \begin{equation*} \begin{gathered} g_{uu}(u,v) = \cos u \sin v \qquad \forall (u,v) \in D, \\ g_{uv}(u,v) = g_{vu}(u,v) = \sin u \cos v \qquad \forall (u,v) \in D, \\ g_{uu}(u,v) = -(1-\cos u) \sin v \qquad \forall (u,v) \in D. \end{gathered} \end{equation*}

Purtroppo però si ha

(176)   \begin{equation*} \nabla^2 g(2\pi,v) = \begin{pmatrix} \sin v	& 	0 \\ 0	& 	0 \end{pmatrix} \qquad \forall v \in [-2\pi,2\pi], \end{equation*}

ovvero le matrici hessiane sono semidefinite, e dunque occorre studiare la natura di questi punti stazionari con altri metodi.

Uno di essi consiste nello studiare il segno di g all’interno di D. Si ha

(177)   \begin{equation*} \begin{gathered} g(u,v) = (1-\cos u)\sin v \geq 0 \iff (u,v) \in D \colon 2k\pi \leq v \leq (2k+1)\pi \qquad \text{per qualche } k \in \mathbb{Z}, \\ g(u,v) = (1-\cos u)\sin v \leq 0 \iff (u,v) \in D \colon (2k-1)\pi \leq v \leq 2k\pi \qquad \text{per qualche } k \in \mathbb{Z}, \end{gathered} \end{equation*}

in quanto il termine 1- \cos u è non-negativo per ogni u, quindi il segno di g dipende da quello del termine \sin u. La situazione è rappresentata in figura 7, in cui nelle aree rosse g assume segno positivo e nelle aree in azzurro g assume segno negativo. Osserviamo inoltre che g(2\pi,v)=0 per ogni v \in [-2\pi,2\pi].

    \[\quad\]

    \[\quad\]

Figura 7: segni assunti dalla funzione g all’interno del dominio D. In rosso le aree in cui g assume segno positivo, in blu le aree dove g assume segno negativo, in nero sono rappresentate le linee su cui g si annulla.

    \[\quad\]

    \[\quad\]

Basandoci su queste informazioni possiamo affrontare con successo lo studio dei punti stazionari di g del tipo (2\pi,v_0) con v_0 \in [-2\pi,2\pi]. Abbiamo due casi.

    \[\quad\]

  • Se v_0 \notin \{0,\pi,2\pi\}, allora g(2\pi,v_0)=0; inoltre da (177) e dalla figura 7 si ottiene che g \geq 0 in un intorno di (2\pi,v_0) (oppure g \leq 0 in un intorno di (2\pi,v_0)). Dunque (2\pi,v) è di minimo relativo se 2k\pi < v < (2k+1)\pi con k \in \{-1,0\}, mentre è di massimo relativo se (2k-1)\pi < v < 2k\pi con k \in \{0,1\}.
  •  

  • Se invece v_0 \in \{0,\pi,2\pi\}, come si vede dalla figura 7, il punto non è né di massimo né di minimo relativo per g in quanto in ogni suo intorno g assume sia segno positivo che negativo.

4. Estremi locali su \partial D. Studiamo ora l’eventuale presenza di punti di estremo relativo al bordo \partial D di D. Si ha

(178)   \begin{equation*} \partial D = \{ (u,v) \in \mathbb{R}^2 \colon v \in \mathbb{R},\,\,v=u\}. \end{equation*}

Se un punto del tipo (u,u) è di estremo relativo per g in D, deve esserlo necessariamente anche per la restrizione di g alla curva di equazione v=u.

L’idea è quindi di ricercare gli eventuali punti di estremo relativo per g su \partial D tra i punti di estremo relativo per la funzione h \colon \mathbb{R} \to \mathbb{R} definita da

(179)   \begin{equation*} h(u) = g(u,u) = (1-\cos u)\sin u \qquad \forall u \in \mathbb{R}. \end{equation*}

A tal fine, calcoliamo la derivata di h e studiamone il segno. Si ha

(180)   \begin{equation*} h'(u) = \cos u + \sin^2 u - \cos^2 u = - 2\cos^2 u  + \cos u + 1 \qquad \forall u \in \mathbb{R}, \end{equation*}

dove si è utilizzata la nota relazione \sin^2 u + \cos^2 u = 1. Osservando che, per la periodicità di h, possiamo restringere lo studio del segno di h' all’intervallo [-\pi,\pi], si ha

(181)   \begin{equation*} h'(u) \geq 0 \qquad \forall u \in \left [-\frac{2}{3} \pi , \frac{2}{3}\pi \right ]. \end{equation*}

Dunque i punti \dfrac{2}{3}\pi +2k\pi sono di massimo assoluto per h, mentre i punti -\dfrac{2}{3} \pi +2k\pi sono di minimo assoluto per h. La monotonia di h (e quindi di g) in un intorno del punto \left ( \dfrac{2}{3}\pi, \dfrac{2}{3} \pi \right ) è rappresentata dalle frecce in verde in figura 8.

Ritornando a g, ciò implica che eventuali punti di massimo relativo per g su \partial D vanno cercati tra

(182)   \begin{equation*} \left ( \frac{2}{3} \pi +2k\pi , \frac{2}{3} \pi +2k\pi \right ) \qquad \forall k \in \mathbb{Z}, \end{equation*}

mentre eventuali punti di minimo relativo per g su \partial D vanno cercati tra

(183)   \begin{equation*} \left ( -\frac{2}{3} \pi +2k\pi, -\frac{2}{3} \pi +2k\pi \right ) \qquad \forall k \in \mathbb{Z}. \end{equation*}

Per la periodicità di g e per il fatto che g è pari rispetto a u e dispari rispetto a v, è sufficiente studiare la natura del punto \left (\dfrac{2}{3} \pi , \dfrac{2}{3} \pi \right ), in quanto quella del punto \left (-\dfrac{2}{3} \pi , -\dfrac{2}{3} \pi \right ) sarà opposta.

    \[\quad\]

    \[\quad\]

Figura 8: in verde, la monotonia della restrizione di g a \partial D, mentre in blu è raffigurata la monotonia della restrizione di g alla semiretta S in un intorno del punto \left (\dfrac{2}{3} \pi , \dfrac{2}{3} \pi \right ). Si vede che la prima ha un massimo in \left (\dfrac{2}{3} \pi , \dfrac{2}{3} \pi \right ), mentre la seconda assume valori strettamente maggiori in ogni intorno di \left (\dfrac{2}{3} \pi , \dfrac{2}{3} \pi \right ). Quindi tale punto non è né di massimo né di minimo per g.

    \[\quad\]

    \[\quad\]

Affermiamo che il punto \left (\dfrac{2}{3} \pi , \dfrac{2}{3} \pi \right ) non è né di massimo né di minimo relativo per g, in quanto restringendo g alla semiretta S definita da \left (\dfrac{2}{3} \pi + t, \dfrac{2}{3} \pi - t \right ) per t \geq 0 (rappresentata in grigio in figura 8 e che è inclusa in D), si ottiene la funzione \varphi \colon [0,+\infty) \to \mathbb{R} definita da

(184)   \begin{equation*} \varphi(t) = g\left (\frac{2}{3} \pi + t, \frac{2}{3} \pi - t \right ) = \left ( 1 - \cos \left(  \frac{2}{3} \pi + t \right ) \right )\sin \left ( \frac{2}{3} \pi - t \right ) \qquad \forall t \in [0,+\infty). \end{equation*}

Osserviamo che \varphi, per come è definita, è pari alla composizione g \circ \gamma, dove \gamma \colon [0,+\infty) \to \mathbb{R} è la funzione definita da

(185)   \begin{equation*} \gamma(t) = \left (\frac{2}{3} \pi + t, \frac{2}{3} \pi - t \right ) \qquad \forall t \in [0,+\infty), \end{equation*}

ossia è una parametrizzazione della semiretta S. Per la regola della catena [5, teorema 3], oppure più semplicemente per la formula delle derivate direzionali di una funzione di più variabili, si ha

(186)   \begin{equation*} \begin{split} \varphi'(t) = & \nabla g\left (\frac{2}{3} \pi + t, \frac{2}{3} \pi - t \right ) \cdot \gamma'(t) \\ = & \nabla g\left (\frac{2}{3} \pi + t, \frac{2}{3} \pi - t \right ) \cdot (1,-1) \\ = & \sin \left (\frac{2}{3} \pi + t \right ) \sin \left (\frac{2}{3} \pi - t \right ) -  \left ( 1- \cos \left (\frac{2}{3} \pi + t \right ) \right ) \cos \left (\frac{2}{3} \pi - t \right ) \qquad \forall t \geq 0 \end{split} \end{equation*}

Osserviamo che \varphi' è una funzione continua e che

(187)   \begin{equation*} \varphi'(0) = \sin \left (\frac{2}{3} \pi \right ) \sin \left (\frac{2}{3} \pi  \right ) -  \left ( 1- \cos \left (\frac{2}{3} \pi \right ) \right ) \cos \left (\frac{2}{3} \pi \right ) = \frac{3}{4} + \frac{3}{4} > 0. \end{equation*}

Quindi esiste \delta>0 tale che

(188)   \begin{equation*} \varphi'(t) > 0  \qquad \forall t \in (0,\delta), \end{equation*}

ovvero tale che la funzione \varphi, cioè la restrizione di g alla semiretta S, è strettamente crescente. Tale monotonia è rappresentata dalle frecce in blu in figura 8.

Questa monotonia implica che g assume dei valori maggiori di g \left (\dfrac{2}{3} \pi , \dfrac{2}{3} \pi \right ) in ogni intorno del punto \left (\dfrac{2}{3} \pi , \dfrac{2}{3} \pi \right ). Poiché invece il punto \left (\dfrac{2}{3} \pi , \dfrac{2}{3} \pi \right ) è di massimo per la restrizione di g al bordo di D, ciò implica che in ogni intorno di \left (\dfrac{2}{3} \pi , \dfrac{2}{3} \pi \right ) la funzione g assume sia valori maggiori che minori di g \left (\dfrac{2}{3} \pi , \dfrac{2}{3} \pi \right ). Quindi il punto \left (\dfrac{2}{3} \pi , \dfrac{2}{3} \pi \right ) non è né di massimo, né di minimo relativo per g.

5. Risultati per f. Ricapitolando, e tornando a f, si ha che i punti di massimo assoluto per f sono tutti e soli i punti (x,y) \in \mathbb{R}^2 che soddisfano

(189)   \begin{equation*} x^2 - y=\pi + 2h\pi, \quad 2x-y - 1 = \frac{\pi}{2} + 2k\pi, \qquad \text{per qualche } h, k \in \mathbb{Z},\,\, k \leq h \end{equation*}

mentre i punti di minimo assoluto per f sono tutti e soli i punti (x,y) \in \mathbb{R}^2 che soddisfano

(190)   \begin{equation*} x^2 - y= \pi + 2h\pi, \quad 2x-y - 1 = \frac{3}{2} \pi + 2k\pi, \qquad \text{per qualche } h, k \in \mathbb{Z}, \,\, k \leq h-1. \end{equation*}

Inoltre i punti di massimo locale per f sono tutti e soli quelli di massimo assoluto insieme ai punti (x,y) \in \mathbb{R}^2 che soddisfano

(191)   \begin{equation*} x^2 - y=2h\pi, \quad 2x-y - 1\in \big( (2k-1)\pi, 2k\pi \big), \qquad \text{per qualche } h, k \in \mathbb{Z},\,\, k \leq h \end{equation*}

mentre i punti di minimo relativo per f sono tutti e soli quelli di minimo assoluto insieme ai punti (x,y) \in \mathbb{R}^2 che soddisfano

    \begin{equation*} x^2 - y= 2h\pi, \quad 2x-y - 1  \in \big( 2k\pi, (2k+1)\pi \big) \qquad \text{per qualche } h, k \in \mathbb{Z}, \,\, k \leq h-1.  \end{equation*}

Osservazione 3.1. Per studiare la presenza di punti di massimo o minimo relativo per g su \partial D, si poteva anche osservare che questi corrispondono all’insieme dei punti (x,y) \in \mathbb{R}^2 tali che

(192)   \begin{equation*} v=u \iff x^2 - y = 2x - y - 1 \iff (x - 1)^2 = 0 \iff x=1. \end{equation*}

Quindi si sarebbe potuto anche studiare la presenza di massimi e minimi relativi per f sulla retta di equazione x=1; tale studio si poteva condurre in maniera analoga a quanto svolto nel caso del cambio di variabili.

 

Scarica gli esercizi svolti

Ottieni il documento contenente 16 esercizi svolti sul calcolo dei massimi e minimi quando il determinante della matrice hessiana è nullo.

 

Esercizio 11  (\bigstar\bigstar\bigstar\largewhitestar\largewhitestar). Determinare estremi assoluti e locali della funzione f \colon \big(\mathbb{R}\setminus \{0\}\big)^2 \to \mathbb{R} definita da

(193)   \begin{equation*} f(x,y) = \frac{x}{y} + \frac{y}{x} \qquad \forall (x,y) \in \big(\mathbb{R}\setminus \{0\}\big)^2. \end{equation*}

Svolgimento.

Osserviamo innanzitutto che la funzione f è dispari rispetto a x e rispetto a y, pertanto possiamo limitarci a studiarla nel primo quadrante (0,+\infty)^2. Osserviamo che, restringendoci alla parabola di equazione y=x^2, si ha

(194)   \begin{equation*} \lim_{x \to + \infty} f(x,x^2) = \lim_{x \to + \infty} \left ( \frac{x}{x^2} + \frac{x^2}{x} \right ) = \lim_{x \to + \infty} \left ( \frac{1}{x} + x \right ) = +\infty. \end{equation*}

Da ciò e dal fatto che f è dispari rispetto alle variabili, si ottiene

(195)   \begin{equation*} \sup_{\big(\mathbb{R}\setminus \{0\}\big)^2} f = +\infty, \qquad \inf_{\big(\mathbb{R}\setminus \{0\}\big)^2} f = -\infty. \end{equation*}

Per studiare eventuali punti di massimo e minimo relativi per f, notiamo che f \in \mathcal{C}^{\infty}\big((\mathbb{R}\setminus \{0\})^2\big), perché somma di funzioni razionali, e utilizziamo il teorema 1.9 ricercandoli tra i suoi punti stazionari. A tal fine, studiamo le derivate parziali:

(196)   \begin{equation*} \begin{gathered} f_x(x,y) = \frac{1}{y} - \frac{y}{x^2} \qquad \forall (x,y) \in \big(\mathbb{R}\setminus \{0\}\big)^2, \\ f_y(x,y) = - \frac{x}{y^2} + \frac{1}{x} \qquad \forall (x,y) \in \big(\mathbb{R}\setminus \{0\}\big)^2. \end{gathered} \end{equation*}

Per i punti stazionari di f si ottiene

(197)   \begin{equation*} \begin{split} \nabla f(x,y) = (0,0) \iff & \begin{cases} \dfrac{1}{y} - \dfrac{y}{x^2}=0 \\[8pt] - \dfrac{x}{y^2} + \dfrac{1}{x} = 0 \end{cases} \\[6pt] \iff & x^2=y^2 \end{split} \end{equation*}

Restringendoci al primo quadrante, otteniamo quindi una semiretta di punti stazionari di equazione x=y.

Per determinare la natura di tali punti stazionari, studiamo preliminarmente le derivate seconde di f al fine di applicare il teorema 1.10:

(198)   \begin{equation*} \begin{gathered} f_{xx}(x,y) = \dfrac{2y}{x^3} \qquad \forall (x,y) \in \big(\mathbb{R}\setminus \{0\}\big)^2, \\[4pt] f_{xy}(x,y) = f_{yx}(x,y) = - \dfrac{1}{y^2} - \dfrac{1}{x^2} \qquad \forall (x,y) \in \big(\mathbb{R}\setminus \{0\}\big)^2, \\[4pt] f_{yy}(x,y) = \dfrac{2x}{y^3} \qquad \forall (x,y) \in \big(\mathbb{R}\setminus \{0\}\big)^2. \end{gathered} \end{equation*}

Per ognuno dei punti sulla retta x=y si ha

(199)   \begin{equation*} \nabla^2 f(x,x) = \begin{pmatrix} \dfrac{2}{x^2} 		& -\dfrac{2}{x^2}\\[8pt] -\dfrac{2}{x^2} 		& \dfrac{2}{x^2} \end{pmatrix} \quad \Longrightarrow \quad \det \nabla^2 f(x,x) = 0 \qquad \forall x \in (0,+\infty), \end{equation*}

dunque la matrice hessiana è semidefinita. Poiché la sua traccia è positiva, essa è semidefinita positiva per la proposizione 1.13, dunque i punti sulla retta x=y sono di sella o di minimo relativo per il teorema 1.10. Per determinarne la natura, possiamo scrivere f nel seguente modo:

(200)   \begin{equation*} f(x,y) = \frac{x^2+y^2}{xy} \qquad \forall (x,y) \in \big(\mathbb{R}\setminus \{0\}\big)^2. \end{equation*}

Fissiamo (x_0,x_0) con x_0>0. Poiché (x_0,x_0) si trova all’interno del primo quadrante, possiamo considerare che in un suo intorno U si abbia x>0 e y>0. Possiamo utilizzare la disuguaglianza

(201)   \begin{equation*} x^2+y^2 \geq 2xy \qquad \forall (x,y) \in \mathbb{R}^2 \end{equation*}

e ottenere che

(202)   \begin{equation*} f(x,y) = \frac{x^2+y^2}{xy} \geq \frac{2xy}{xy} = 2 \qquad \forall (x,y) \in U. \end{equation*}

Poiché f(x_0,y_0)=2, tale disuguaglianza mostra che (x_0,y_0) è un punto di minimo relativo per f.

Dato che f è dispari rispetto a entrambe le variabili, risulta che la retta di punti (x,x), con x \neq 0 è costituita da punti di minimo relativo per f, mentre la retta dei punti (x,-x) con x \neq 0 è costituita da punti di massimo relativo per f.


Esercizio 12  (\bigstar\bigstar\bigstar\bigstar\largewhitestar). Determinare estremi assoluti e locali della funzione f \colon \mathbb{R}^2 \to \mathbb{R} definita da

(203)   \begin{equation*} f(x,y) = y^2 \left ( x^6 -2y^2 +y^4 \right ) \qquad \forall (x,y) \in \mathbb{R}^2. \end{equation*}

Svolgimento.

Vale

(204)   \begin{equation*} \lim_{y \to \pm \infty} f(0,y) = \lim_{y \to \pm \infty} y^2(-2y^2+y^4) = \lim_{y \to \pm \infty} y^4(y^2-y) = +\infty, \end{equation*}

da cui segue che

(205)   \begin{equation*} \sup_{\mathbb{R}^2} f = +\infty. \end{equation*}

Affermiamo che f ha minimo assoluto su \mathbb{R}^2. Infatti, innanzitutto osserviamo che se x^2+y^2 \geq 4, allora deve aversi x^2\geq 2 oppure y^2\geq 2. Nel primo caso si ha x^6\geq 8 e dunque

(206)   \begin{equation*} x^6-2y^2+y^4 \geq 8 - 2 + 4>0. \end{equation*}

Nel secondo caso si ha

(207)   \begin{equation*} x^6-2y^2+y^4 \geq  -4+4 = 0. \end{equation*}

Abbiamo dunque mostrato che

(208)   \begin{equation*} f(x,y) \geq 0 \qquad \forall (x,y) \in \mathbb{R}^2 \colon x^2+y^2 \geq 4. \end{equation*}

Consideriamo ora il cerchio D di centro (0,0) e di raggio 2; esso è compatto poiché chiuso e limitato e f è di classe \mathcal{C}^{\infty}(\mathbb{R}^2) poiché è polinomiale. Quindi f ha minimo su D per il teorema di Weierstrass 1.8. Inoltre, dato che

(209)   \begin{equation*} f(0,1) = -1 < 0, \end{equation*}

si ha che \min_D f \leq -1 e, per (208), abbiamo

(210)   \begin{equation*} \min_D f = \min_{\mathbb{R}^2} f, \end{equation*}

e dunque il minimo di f in \mathbb{R}^2 esiste.

Per trovare i punti di minimo e trovare altri eventuali estremi relativi di f, usiamo il teorema 1.9 e determiniamo i punti stazionari di f. Cominciamo col calcolare le derivate parziali di f:

(211)   \begin{equation*} \begin{gathered} f_x(x,y) = 6x^5y^2 \qquad \forall (x,y) \in \mathbb{R}^2, \\ f_y(x,y) = 2yx^6 - 8y^3 + 6y^5 \qquad \forall (x,y) \in \mathbb{R}^2. \end{gathered} \end{equation*}

Calcoliamo ora i punti stazionari di f:

(212)   \begin{equation*} \begin{split} \nabla f(x,y) = (0,0) \iff & \begin{cases} x=0 \vee y=0 \\ 2yx^6 - 8y^3 + 6y^5 = 0 \end{cases} \\ \iff & y=0 \vee \begin{cases} x=0 \\ - 8y^3 + 6y^5 = 0 \end{cases} \\ \iff & y=0 \vee \begin{cases} x=0 \\ y = \pm \dfrac{2}{\sqrt{3}}. \end{cases} \end{split} \end{equation*}

Da ciò segue dunque che i punti stazionari sono quelli sull’asse x insieme ai due punti

(213)   \begin{equation*} \left ( 0, \frac{2}{\sqrt{3}} \right ), \quad \left ( 0, -\frac{2}{\sqrt{3}} \right ). \end{equation*}

Per determinarne la natura proviamo a usare il teorema 1.10 e studiamo le derivate seconde di f:

(214)   \begin{equation*} \begin{gathered} f_{xx}(x,y) = 30x^4y^2 \qquad \forall (x,y) \in \mathbb{R}^2, \\[4pt] f_{xy}(x,y) = f_{yx}(x,y) = 12x^5y \qquad \forall (x,y) \in \mathbb{R}^2, \\[4pt] f_{yy}(x,y) = 2x^6 - 24y^2 + 30y^4 \qquad \forall (x,y) \in \mathbb{R}^2. \end{gathered} \end{equation*}

Studieremo separatamente l’origine dagli altri punti dell’asse x in quanto, a posteriori, si vedrà che la loro natura è diversa.

\bullet (0,0). Osserviamo che nell’origine la matrice hessiana è nulla:

(215)   \begin{equation*} \nabla^2 f(0,0) = \begin{pmatrix} 0	& 	0\\ 0	&	0 \end{pmatrix}, \end{equation*}

dunque il teorema 1.10 non fornisce informazioni, per cui studiamo la natura di tale punto stazionario con considerazioni di carattere diverso. Osserviamo che si ha

(216)   \begin{equation*} f(0,y) = y^2(-2y^2 + y^4) = y^4( -2 + y^2) < 0 \qquad \forall y \in (-\sqrt{2}, \sqrt{2}). \end{equation*}

D’altra parte, restringendoci alla curva di equazione y=\frac{1}{2} x^3 si ha

(217)   \begin{equation*} f(x,x^3) = \frac{1}{4} x^6 \left ( x^6 - \frac{x^6}{2} + \frac{x^{12}}{16} \right ) = \frac{x^{12}}{8} \left( 1 - \frac{x^6}{8} \right ) > 0 \qquad \forall x \in (-1,0) \cup (0,1). \end{equation*}

Poiché da (216) e (217) f assume valori di segno opposto in ogni intorno di (0,0), tale punto è di sella per f.

Notiamo che il fatto che f assuma segno positivo in ogni intorno di (0,0) non poteva essere ottenuto studiando la restrizione di f a rette sull’origine. Invitiamo il lettore a verificare che la restrizione di f a una qualunque di queste rette assume segno negativo in un intorno sufficientemente piccolo dell’origine.

\bullet (x_0,0) con x_0 \in \mathbb{R} \setminus \{0\}. Per gli altri punti stazionari appartenenti all’asse x si ha

(218)   \begin{equation*} \nabla^2 f(x_0,0) = \begin{pmatrix} 0	& 	0\\ 0	&	0 \end{pmatrix} \qquad \forall x_0 \in \mathbb{R} \setminus \{0\}, \end{equation*}

dunque la matrice hessiana è nulla e quindi occorre determinare la natura di tali punti con metodi diversi dal teorema 1.10.

Osserviamo che, poiché x_0^6>0 e la funzione (x,y) \mapsto x^6-2y^2+y^4 è continua, esiste un intorno U di (x_0,0) tale che

(219)   \begin{equation*} x^6-2y^2+y^4>0 \qquad \forall (x,y) \in U. \end{equation*}

Da ciò segue che

(220)   \begin{equation*} f(x,y) \geq 0 \qquad \forall (x,y) \in U. \end{equation*}

Dato che f(x_0,0)=0, da tale disuguaglianza si ha che (x_0,0) è un punto di minimo relativo per f.

\bullet \left ( 0, \dfrac{2}{\sqrt{3}} \right ). Per la parità di f rispetto a y, la natura di \left ( 0, \dfrac{2}{\sqrt{3}} \right ) e di \left ( 0, -\dfrac{2}{\sqrt{3}} \right ) è la stessa. Inoltre sappiamo dalla discussione iniziale sugli estremi assoluti che f deve possedere almeno un punto di minimo assoluto e che tale valore minimo deve essere negativo. Per il teorema di Fermat 1.9, i punti di minimo assoluto per f sono stazionari e, per esclusione, ciò implica che questi devono essere proprio

(221)   \begin{equation*} \left ( 0, \pm \frac{2}{\sqrt{3}} \right ), \end{equation*}

in quanto negli altri punti stazionari studiati la funzione f si annulla, mentre sappiamo che \min_{\mathbb{R}^2} f < 0. Dunque \left ( 0, \pm \dfrac{2}{\sqrt{3}} \right ) sono punti di minimo assoluto per f.

Se si volesse analizzare la natura di questi punti con un argomento diretto, osserviamo che la matrice hessiana soddisfa

(222)   \begin{equation*} \nabla^2 f \left ( 0, \frac{2}{\sqrt{3}} \right ) = \begin{pmatrix} 0	& 	0\\[5pt] 0	&	-8 + \dfrac{160}{3} \end{pmatrix}, \end{equation*}

che è semidefinita positiva in quanto possiede un autovalore nullo e uno positivo. Dunque tali punti possono essere di minimo relativo o di sella per f per il teorema 1.10. Per determinarlo con esattezza, studiamo ad esempio le derivate parziali prime di f. Fissiamo (x_0,y_0) tale che y_0>0.

Studiando la derivata parziale rispetto a y, otteniamo

(223)   \begin{equation*} f_y(0,y) = -8y^3+6y^5 = 2y^3(3y^2-4) \quad \begin{cases} >0 \text{ per } y \in \left ( \dfrac{2}{\sqrt{3}} ,y_0\right )		& \text{se } y_0 > \dfrac{2}{\sqrt{3}} \\[8pt] <0 \text{ per } y \in \left ( y_0,  \dfrac{2}{\sqrt{3}} \right )		& \text{se } y_0 \in \left ( 0, \dfrac{2}{\sqrt{3}} \right ), \end{cases} \end{equation*}

precisando che tali conclusioni sono valide se y_0>0. Come rappresentato in figura 9, ciò implica che

(224)   \begin{equation*} f(0,y_0) \geq f\left (0, \frac{2}{\sqrt{3}} \right ) \qquad \text{se } y_0>0 \end{equation*}

e tale disuguaglianza è stretta se y_0 \neq \dfrac{2}{\sqrt{3}}. Si ha poi

(225)   \begin{equation*} f_x(x,y_0) = 6x^5y_0^2 >0 \qquad \forall x >0, \end{equation*}

dunque la restrizione di f ai segmenti orizzontali rappresentati in figura 9 è crescente.

    \[\quad\]

    \[\quad\]

Figura 9: rappresentazione della monotonia della funzione f sulle rette di equazione x=0 e y=y_0 con y_0>0. Le frecce indicano la direzione in cui f è crescente ottenuta studiando rispettivamente il segno di f_x e f_y. Da queste monotonie si ottiene che il punto \left( 0, \dfrac{2}{\sqrt{3}} \right ) è di minimo relativo per f.

    \[\quad\]

    \[\quad\]

Si ha quindi

(226)   \begin{equation*} f(x_0,y_0) \geq f(0,y_0) \end{equation*}

e di nuovo la disuguaglianza è stretta se x_0>0. Da ciò e da (224) deduciamo che

(227)   \begin{equation*} f(x_0,y_0) \geq f(0,y_0) \geq f\left (0, \frac{2}{\sqrt{3}} \right ), \end{equation*}

e almeno una delle disuguaglianze è stretta se (x_0,y_0) \neq \left (0, \dfrac{2}{\sqrt{3}}\right ). Ciò mostra che \left (0, \dfrac{2}{\sqrt{3}}\right ) è un punto di minimo relativo per f.


 

Esercizio 13  (\bigstar\bigstar\bigstar\largewhitestar\largewhitestar). Determinare estremi assoluti e locali della funzione f \colon \mathbb{R}^2 \to \mathbb{R} definita da

(228)   \begin{equation*} f(x,y) = -2\cos y + x^2+ 2x\sin y \qquad \forall (x,y) \in \mathbb{R}^2. \end{equation*}

Svolgimento.

Osserviamo innanzitutto che la funzione è periodica di periodo 2\pi rispetto a y, dunque possiamo limitarci a studiarla nella striscia

(229)   \begin{equation*} D= \{ (x,y) \in \mathbb{R}^2 \colon 0 \leq y \leq 2\pi\}. \end{equation*}

Restringendo f alla retta y=0, si ha

(230)   \begin{equation*} \lim_{x \to \pm \infty} f(x,0) = \lim_{x \to \pm \infty} (-2 + x^2) = +\infty, \end{equation*}

dunque

(231)   \begin{equation*} \sup_{\mathbb{R}^2} f = +\infty. \end{equation*}

Osserviamo poi che

(232)   \begin{equation*} f(x,y) \geq  -2\cos y + x^2  - 2|x| = -2\cos y +  \left( |x|- 1 \right )^2 - 1 \geq - 3 + \left( |x|- 1 \right )^2 \geq 1 \qquad \forall (x,y) \in D \colon |x|>3. \end{equation*}

Dato che f(0,0)=-2, l’estremo inferiore di f va cercato nella striscia di piano |x|\leq 3, ovvero

(233)   \begin{equation*} \inf_{(x,y) \in \mathbb{R}^2} f(x,y) = \inf_{(x,y) \in D \colon |x|\leq 3} f(x,y). \end{equation*}

Da ciò, tenendo conto che f è continua e che il rettangolo Q = [-3,3] \times [0,2\pi] è compatto, per il teorema di Weierstrass 1.8 si ha

(234)   \begin{equation*} \inf_{\mathbb{R}^2} f = \min_{Q} f, \end{equation*}

ossia f possiede minimo assoluto. Per determinare tale minimo e altri eventuai estremi relativi di f, che è di classe \mathcal{C}^\infty(\mathbb{R}^2), usiamo il teorema 1.9 e ricerchiamo i punti stazionari di f. Cominciamo col calcolarne le derivate parziali:

(235)   \begin{equation*} \begin{gathered} f_x(x,y) = 2x + 2\sin y \qquad \forall (x,y) \in \mathbb{R}^2, \\ f_y(x,y) = 2\sin y + 2x\cos y \qquad \forall (x,y) \in \mathbb{R}^2. \end{gathered} \end{equation*}

Determiniamo ora i punti stazionari di f:

(236)   \begin{equation*} \begin{split} \nabla f(x,y) = (0,0) \iff & \begin{cases} x + \sin y = 0 \\ \sin y + x\cos y = 0 \end{cases} \\ \iff & \begin{cases} x = - \sin y \\ x = x \cos y \end{cases} \\ \iff & \begin{cases} x = - \sin y \\ x=0 \,\, \vee \,\, \cos y=1 \end{cases} \\ \iff & \begin{cases} \sin y = 0 \\ x=0 \end{cases} \vee \begin{cases} x = 0 \\ \cos y=1. \end{cases} \end{split} \end{equation*}

Poichè dalla periodicità di f rispetto a y è sufficiente studiare soltanto le soluzioni con y \in [0,2\pi), l’unico punto stazionario da considerare è

(237)   \begin{equation*} (0,0). \end{equation*}

Avevamo già dedotto che f possiede minimo assoluto e, per il teorema di Fermat 1.9, tale minimo assoluto è un suo punto stazionario. Poiché, a meno di multipli di 2\pi nella variabile y, (0,0) è l’unico punto stazionario di f, esso deve essere il punto di minimo assoluto.

Se volessimo determinare la natura di (0,0) direttamente, senza utilizzare l’unicità del punto stazionario, proviamo a calcolare le derivate seconde di f per applicare il teorema 1.10:

(238)   \begin{equation*} \begin{gathered} f_{xx}(x,y) = 2 \qquad \forall (x,y) \in \mathbb{R}^2, \\[4pt] f_{xy}(x,y) = f_{yx}(x,y) = 2\cos y \qquad \forall (x,y) \in \mathbb{R}^2, \\[4pt] f_{yy}(x,y) = 2\cos y - 2x \sin y \qquad \forall (x,y) \in \mathbb{R}^2. \end{gathered} \end{equation*}

Si ha

(239)   \begin{equation*} \nabla^2 f(0,0) = \begin{pmatrix} 2 	&	2 \\ 2	& 	2 \end{pmatrix} \quad \Longrightarrow \quad \det \nabla^2 f(0,0) = 0, \end{equation*}

ossia \nabla^2 f(0,0) è semidefinita. Più precisamente, essa è semidefinita positiva in quanto la sua traccia è pari a 4, quindi dalla proposizione 1.13 essa possiede un autovalore nullo e uno pari a 4. Dunque per il teorema 1.10 il punto (0,0) può essere di sella o di minimo relativo per f, ma occorre usare altri metodi per stabilire se si ricada nell’uno o nell’altro caso. Osserviamo che

(240)   \begin{equation*} \begin{split} f(x,y) = & x^2+2x\sin y - 2 \cos y \\ = & x^2 + 2x \sin y + \sin^2 y - \sin^2 y - 2 \cos y \\ = & (x+ \sin y)^2 + 1- \sin^2 y - 1 - 2 \cos y \\ = & (x+ \sin y)^2 + \cos^2 y - 2 \cos y + 1 - 2 \\ = & (x+ \sin y)^2 + (\cos y - 1)^2 - 2 \qquad \forall (x,y) \in \mathbb{R}^2. \end{split} \end{equation*}

Da tale relazione, poiché f è pari alla somma del numero -2 con due termini quadratici, segue che

(241)   \begin{equation*} f(x,y) \geq -2 \qquad \forall (x,y) \in \mathbb{R}^2, \end{equation*}

ovvero che (0,0) è un punto di minimo (assoluto) per f. Notiamo esplicitamente che questa scrittura di f permetteva immediatamente di notare che f è limitata inferiormente da -2. Inoltre si poteva immediatamente osservare che, poiché tale valore è assunto nei punti (0,2k\pi) con k \in \mathbb{Z}, questi punti sono dunque di minimo assoluto per f.


 

Esercizio 14  (\bigstar\bigstar\bigstar\bigstar\largewhitestar). Determinare estremi assoluti e locali della funzione f \colon \mathbb{R}^2 \to \mathbb{R} definita da

(242)   \begin{equation*} f(x,y) = 5+(x+\sin y)^3 (x-\sin y) \qquad \forall (x,y) \in \mathbb{R}^2. \end{equation*}

Svolgimento.

La funzione è periodica di periodo 2\pi rispetto alla variabile y, pertanto è sufficiente studiarla nella striscia

(243)   \begin{equation*} D \coloneqq \{ (x,y) \in \mathbb{R}^2 \colon 0 \leq y \leq 2\pi \}. \end{equation*}

Restringendoci alla retta y=0 si ha

(244)   \begin{equation*} \lim_{x \to \pm \infty} f(x,0) = \lim_{x \to \pm \infty} (5 + x^4) = +\infty, \end{equation*}

dunque f è illimitata superiormente, ossia

(245)   \begin{equation*} \sup_{D} f(x,y) = +\infty. \end{equation*}

Osserviamo che

(246)   \begin{equation*} x+ \sin y, \geq 1 \quad x- \sin y \geq 1 \qquad \forall |x|\geq 2, \,\,\forall y \in \mathbb{R}, \end{equation*}

dunque

(247)   \begin{equation*} f(x,y) \geq 5 + 1 = 6 \qquad \forall (x,y) \in \mathbb{R}^2 \colon |x| \geq 2. \end{equation*}

Poiché f(0,0)=5<6, ciò implica che l’estremo inferiore di f va ricercato nel rettangolo

(248)   \begin{equation*} Q \coloneqq \{ (x,y) \in \mathbb{R}^2 \colon -2 \leq x \leq 2,\,\,0 \leq y \leq 2\pi \}. \end{equation*}

Poiché f \in \mathcal{C}^{\infty}(\mathbb{R}^2), per il teorema di Weierstrass 1.8 essa possiede minimo assoluto su Q e quindi

(249)   \begin{equation*} \inf_{\mathbb{R}^2} f = \min_{\mathbb{R}^2} f = \min_Q f \leq 5. \end{equation*}

Per determinare i punti di minimo assoluto ed altri eventuali punti di estremo locale, utilizziamo il teorema 1.9, ricercandoli tra i punti stazionari di f. Calcoliamo le derivate parziali di f:

(250)   \begin{equation*} \begin{gathered} f_x(x,y) = 3(x+\sin y)^2(x-\sin y) + (x+\sin y)^3 %= %(x+\sin y)^2(3x - 3\sin y + x + \sin y) = 2(x+\sin y)^2(2x-\sin y) \qquad \forall (x,y) \in \mathbb{R}^2, \\[7pt] \begin{split} f_y(x,y) = & 3(x+\sin y)^2\cos y(x-\sin y) - (x+\sin y)^3\cos y \\ = & 2(x+\sin y)^2\cos y(x - 2\sin y) \qquad \qquad \forall (x,y) \in \mathbb{R}^2. \end{split} \end{gathered} \end{equation*}

Determiniamo i punti stazionari di f:

(251)   \begin{equation*} \begin{split} \nabla f(x,y) = (0,0) \iff & \begin{cases} (x+\sin y)^2(2x-\sin y) = 0 \\ (x+\sin y)^2\cos y(x - 2\sin y) = 0 \end{cases} \\ \iff & \begin{cases} x = -\sin y \,\, \vee \,\, x= \dfrac{1}{2} \sin y \\[6pt] x = -\sin y \,\, \vee \,\, \cos y= 0 \,\,\vee \,\, x= 2 \sin y \end{cases} \\ \iff & x= -\sin y \,\,\vee\,\, \begin{cases} x =\dfrac{1}{2} \sin y \\[6pt] \cos y=0 \end{cases} \,\,\vee\,\, \begin{cases} x =\dfrac{1}{2} \sin y \\[6pt] x = 2\sin y \end{cases} \end{split} \end{equation*}

Le soluzioni (x,y) \in D del primo sistema sono

(252)   \begin{equation*} \left (\frac{1}{2}, \frac{\pi}{2} \right ), \qquad \left (- \frac{1}{2}, \frac{3}{2}\pi \right ), \end{equation*}

mentre le due equazioni del terzo sistema implicano \sin y=0, dunque le soluzioni sono

(253)   \begin{equation*} (0,0),\qquad (0,\pi), \end{equation*}

che già appartengono alla curva di equazione x=-\sin y. Ricapitolando, i punti stazionari di f in D sono dunque quelli in (252) insieme alla curva di equazione

(254)   \begin{equation*} x=-\sin y \qquad \forall y \in [0,2\pi]. \end{equation*}

Per determinare la natura di tali punti, si potrebbe tentare di usare il teorema 1.10 e quindi studiare le derivate seconde e la matrice hessiana di f.

(255)   \begin{equation*} \begin{gathered} f_{xx}(x,y) = 4(x + \sin y)(2x-\sin y) + 4(x+\sin y)^2 = 12(x+\sin y)x \qquad \forall (x,y) \in \mathbb{R}^2, \\[5pt] \begin{split} f_{xy}(x,y) = f_{yx}(x,y) = & 4\cos y(x+\sin y)(x-2\sin y) + 2(x+\sin y)^2\cos y \\ = & 6\cos y(x+\sin y)(x-\sin y) \qquad \forall (x,y) \in \mathbb{R}^2, \end{split} \\[5pt] \begin{split} f_{yy}(x,y) = & 4(x+\sin y)\cos^2 y (x-2\sin y) -2 \sin y(x+\sin y)^2(x-2\sin y) \\ & \quad -4(x+ \sin y)^2\cos^2y \qquad \forall (x,y) \in \mathbb{R}^2. \end{split} \end{gathered} \end{equation*}

\bullet (x,y) tali che x=-\sin y. La matrice hessiana è nulla in tali punti:

(256)   \begin{equation*} \nabla^2 f (-\sin y,y) = \begin{pmatrix} 0	&	0 \\ 0	& 0 \end{pmatrix} \qquad \forall y \in \mathbb{R}. \end{equation*}

Occorre quindi determinarne la natura in altro modo. Studiamo il segno della funzione g \colon \mathbb{R}^2 \to \mathbb{R} definita da

(257)   \begin{equation*} g(x,y) = f(x,y)-5 = (x+\sin y)^3(x-\sin y) \qquad \forall (x,y) \in \mathbb{R}^2. \end{equation*}

Si ha

(258)   \begin{equation*} g(x,y) <0 \qquad \forall (x,y) \in \mathbb{R}^2 \colon -|\sin y| < x < |\sin y|, \end{equation*}

regione raffigurata in blu nella figura 10. Invece

(259)   \begin{equation*} g(x,y) >0 \qquad \forall (x,y) \in \mathbb{R}^2 \colon x >|\sin y|, \end{equation*}

regione raffigurata in rosso nella figura 10. Poiché g assume segni opposti in un intorno di ogni punto della curva x=-\sin y, tali punti sono di sella per g, e quindi per f.

    \[\quad\]

    \[\quad\]

Figura 10: segni assunti dalla funzione g. In rosso le aree in cui g assume segno positivo, ovvero la regione x > |\sin y|, in blu le aree dove g assume segno negativo, ossia la regione -|\sin y| \leq x \leq |\sin x|. Da ciò si vede che i punti della curva x=-\sin y sono di sella per g.

    \[\quad\]

    \[\quad\]

\bullet \left (\dfrac{1}{2}, \dfrac{\pi}{2} \right ) e \left (-\dfrac{1}{2}, \dfrac{3}{2} \pi \right ). Si ha

(260)   \begin{equation*} \nabla^2 f  \left (\frac{1}{2}, \frac{\pi}{2} \right ) = \nabla^2 f  \left (-\frac{1}{2}, \frac{3}{2}\pi \right ) = \begin{pmatrix} 9	&	0 \\[5pt] 0	& \dfrac{27}{4} \end{pmatrix}, \end{equation*}

dunque tali matrici hessiane sono definite positive, in quanto possiedono i due autovalori positivi 9 e \dfrac{27}{4}. Il teorema 1.10 implica quindi che i punti \left (\dfrac{1}{2}, \dfrac{\pi}{2} \right ) e \left (-\dfrac{1}{2}, \dfrac{3}{2} \pi \right ) sono di minimo relativo per f. Poiché si era già stabilito che f possedesse dei punti di minimo assoluto, e dato che

(261)   \begin{equation*} f\left (\frac{1}{2}, \dfrac{\pi}{2} \right ) = f\left (-\frac{1}{2}, \dfrac{3}{2}\pi \right ), \end{equation*}

questi punti sono anche di minimo assoluto per f.

Alternativamente, invece di studiare le derivate seconde di f, si poteva già dedurre che \left (\dfrac{1}{2}, \dfrac{\pi}{2} \right ) e \left (-\dfrac{1}{2}, \dfrac{3}{2} \pi \right ) sono di minimo assoluto per f dal seguente ragionamento.

Poiché dalla discussione iniziale risulta che f possiede dei punti di minimo assoluto, essi devono essere cercati tra i rimanenti punti stazionari, ossia appunto tra

(262)   \begin{equation*} \left (\frac{1}{2}, \dfrac{\pi}{2} \right ), \quad \left (-\frac{1}{2}, \dfrac{3}{2}\pi \right ). \end{equation*}

Dato che

(263)   \begin{equation*} f\left (\frac{1}{2}, \dfrac{\pi}{2} \right ) = f\left (-\frac{1}{2}, \dfrac{3}{2}\pi \right ), \end{equation*}

e poiché almeno uno di essi è di minimo assoluto per f, ciò implica che lo sono entrambi e che

(264)   \begin{equation*} \min_{\mathbb{R}^2} f = f\left (\frac{1}{2}, \dfrac{\pi}{2} \right ) = 5-\frac{27}{16}. \end{equation*}


 

Esercizio 15  (\bigstar\bigstar\bigstar\largewhitestar\largewhitestar). Si consideri la funzione f \colon \mathbb{R}^2 \to \mathbb{R} definita da

(265)   \begin{equation*} f(x,y) = (\sin x + \sin y)^2 - 4\sin x - 2\sin y \qquad \forall (x,y) \in \mathbb{R}^2. \end{equation*}

Studiare la natura dei punti stazionari di f.

Svolgimento.

Osserviamo preliminarmente che f è periodica di periodo 2\pi rispetto a entrambe le variabili, dunque è sufficiente determinare la natura dei punti critici contenuti nell’insieme

(266)   \begin{equation*} D \coloneqq [0,2\pi) \times [0,2\pi). \end{equation*}

Osserviamo che f è di classe \mathcal{C}^{\infty}(\mathbb{R}^2) in quanto somma e composizione di funzioni \sin. Calcoliamo le derivate parziali di f:

(267)   \begin{equation*} \begin{gathered} f_x = 2(\sin x + \sin y)\cos x - 4 \cos x = 2\cos x( \sin x + \sin y - 2) \qquad \forall (x,y) \in \mathbb{R}^2, \\[5pt] f_y = 2(\sin x + \sin y)\cos y - 2 \cos y = 2\cos y ( \sin x + \sin y - 1) \qquad \forall (x,y) \in \mathbb{R}^2. \end{gathered} \end{equation*}

Determiniamo ora i punti stazionari di f:

(268)   \begin{equation*} \begin{split} \nabla f(x,y) = (0,0) \iff & \begin{cases} \cos x( \sin x + \sin y - 2) = 0 \\ \cos y ( \sin x + \sin y - 1) = 0 \end{cases} \\ \iff & \begin{cases} \cos x = 0 \,\, \vee \,\,  \sin x + \sin y - 2 = 0 \\ \cos y = 0 \,\, \vee \,\,  \sin x + \sin y - 1 = 0 \end{cases} \\ \iff & \begin{cases} \cos x=0 \\[6pt] \cos y=0 \end{cases} \,\,\vee\,\, \begin{cases} \cos x = 0 \\[6pt] \sin x + \sin y - 1 = 0, \end{cases} \,\,\vee\,\, \\[4pt] & \vee \begin{cases} \sin x + \sin y - 2 = 0 \\[6pt] \cos y=0, \end{cases} \,\,\vee\,\, \begin{cases} \sin x + \sin y - 2 = 0 \\[6pt] \sin x + \sin y - 1 = 0. \end{cases} \end{split} \end{equation*}

Poiché l’ultimo sistema in (268) è impossibile, calcoliamo le soluzioni dei primi tre. Il primo possiede le seguenti soluzioni in D:

(269)   \begin{gather*} \left (\frac{\pi}{2},\frac{\pi}{2} \right ), \quad \left (\frac{\pi}{2},\frac{3}{2} \pi \right ), \quad \left (\frac{3}{2}\pi,\frac{\pi}{2} \right ), \quad \left (\frac{3}{2}\pi,\frac{3}{2}\pi \right). \end{gather*}

Per il secondo sistema, da \cos x=0 segue \sin x= \pm 1; il caso \sin x=-1 è da escludere perché produrrebbe \sin y-2=0 nella seconda equazione. Allora si ha \sin x=1 e la seconda equazione implica \sin y=0, da cui segue che le soluzioni del secondo sistema appartenenti a D sono

(270)   \begin{equation*} \left (\frac{\pi}{2},0 \right ), \quad \left (\frac{\pi}{2},\pi \right ). \end{equation*}

Analogamente, nel terzo sistema \cos y=0 implica \sin y=\pm 1; il caso \sin y=-1 è da escludere poiché implicherebbe \sin x - 3=0 nella prima equazione, dunque segue che \sin y=1 e quindi anche \sin x=1. Di conseguenza, l’unica soluzione in D del terzo sistema è

(271)   \begin{equation*} \left (\frac{\pi}{2},\frac{\pi}{2} \right ). \end{equation*}

Confrontando le soluzioni dei 3 sistemi, si vede che i punti stazionari di f in D sono tutti e soli i seguenti:

(272)   \begin{equation*} \left (\frac{\pi}{2},0 \right ), \quad \left (\frac{\pi}{2},\frac{\pi}{2} \right ), \quad \left (\frac{\pi}{2},\pi \right ), \quad \left (\frac{\pi}{2},\frac{3}{2} \pi \right ), \quad \left (\frac{3}{2}\pi,\frac{\pi}{2} \right ), \quad \left (\frac{3}{2}\pi,\frac{3}{2}\pi \right). \end{equation*}

Per studiarne la natura, ricaviamo le derivate seconde di f per usare il teorema 1.10:

(273)   \begin{equation*} \begin{gathered} f_{xx}(x,y) = -2\sin x(\sin x + \sin y - 2) + 2\cos^2 x \qquad \forall (x,y) \in \mathbb{R}^2, \\[3pt] f_{xy}(x,y) =  f_{yx}(x,y) = 2\cos x \cos y \qquad \forall (x,y) \in \mathbb{R}^2, \\[3pt] \begin{split} f_{yy}(x,y) -2\sin y(\sin x + \sin y - 1) + 2\cos^2 y \qquad \forall (x,y) \in \mathbb{R}^2. \end{split} \end{gathered} \end{equation*}

\bullet \left (\dfrac{\pi}{2},0 \right ). La matrice hessiana vale

(274)   \begin{equation*} \nabla^2 f \left (\frac{\pi}{2},0\right ) = \begin{pmatrix} 2	&	0 \\ 0	& 2 \end{pmatrix}, \end{equation*}

che è quindi definita positiva poiché diagonale con autovalore 2 di molteplicità pari a 2. Il teorema 1.10 assicura quindi che tale punto è di minimo relativo per f.

\bullet \left (\dfrac{\pi}{2},\dfrac{\pi}{2} \right ). La matrice hessiana in tale punto è pari a

(275)   \begin{equation*} \nabla^2 f \left (\frac{\pi}{2},\frac{\pi}{2} \right ) = \begin{pmatrix} 0	&	0 \\ 0	& -2 \end{pmatrix}, \end{equation*}

che è semidefinita negativa. Osserviamo che la restrizione di f alla retta di equazione y=\dfrac{\pi}{2} ha un minimo locale per x=\dfrac{\pi}{2}, infatti vale

(276)   \begin{equation*} \begin{split} f\left ( \frac{\pi}{2}+t, \frac{\pi}{2} \right ) = g(t) = & \left ( \sin \left ( t + \frac{\pi}{2} \right ) + 1\right )^2 - 4 \sin \left ( t + \frac{\pi}{2} \right ) - 2 \\ = & (\cos t + 1)^2 - 4 \cos t - 2 \\ = & \left (1 - \frac{1}{2}t^2 + \frac{1}{24}t^4 +  o(t^4) + 1 \right )^2 - 4 \left ( 1-\frac{1}{2}t^2 +  \frac{1}{24}t^4  +o(t^4) \right ) - 2 \\ = & 4 - 2t^2 + \frac{1}{4}t^4 + \frac{1}{6}t^4 -4 + 2t^2 - \frac{1}{6} t^4  - 2+ o(t^4) \\ = & - 2 + \frac{1}{4} t^4 + o(t^4) \\ =& - 2 + \frac{1}{4}t^4\big(1+o(1) \big) , \end{split} \end{equation*}

dove alla seconda uguaglianza e alla terza uguaglianza abbiamo usato rispettivamente l’identità e lo sviluppo di Taylor

(277)   \begin{equation*} \sin \left ( t + \frac{\pi}{2} \right )= \cos t, \qquad \cos t= 1-\frac{1}{2}t^2 + \frac{1}{24}t^4 + o(t^4). \end{equation*}

Da (276), poiché 1+o(1) >0 per t \to 0, si vede che la restrizione di f alla retta di equazione y=\dfrac{\pi}{2} ha un minimo locale in x=\dfrac{\pi}{2}. La situazione è rappresentata in verde nella figura 11.

Questa conclusione poteva anche essere ottenuta studiando il segno della derivata prima di g in un intorno di t=0, oppure scrivendo

(278)   \begin{equation*} \begin{split} f\left ( \frac{\pi}{2}+t, \frac{\pi}{2} \right ) = g(t) = & \left ( \sin \left ( t + \frac{\pi}{2} \right ) + 1\right )^2 - 4 \sin \left ( t + \frac{\pi}{2} \right ) - 2 \\ = & (\cos t + 1)^2 - 4 \cos t - 2 \\ = & \cos^2 t - 2 \cos t - 1 \\ = & (\cos t - 1)^2 - 2 \qquad \forall t \in \mathbb{R}, \end{split} \end{equation*}

da cui risulta chiaramente che g ha un minimo assoluto per t=0.

    \[\quad\]

    \[\quad\]

Figura 11: rappresentazione della monotonia delle restrizioni di f alle rette di equazione y=\dfrac{\pi}{2} (in verde) e x=\dfrac{\pi}{2} (in blu). Le frecce rappresentano il fatto che la restrizione g di f alla retta orizzontale presenta un punto di minimo in corrispondenza di \left( \dfrac{\pi}{2}, \dfrac{\pi}{2} \right ), mentre la restrizione h di f alla retta verticale presenta un punto di massimo in corrispondenza di \left( \dfrac{\pi}{2}, \dfrac{\pi}{2} \right ). Per tali ragioni, \left( \dfrac{\pi}{2}, \dfrac{\pi}{2} \right ) è un punto di sella per f.

    \[\quad\]

    \[\quad\]

D’altra parte, la restrizione di f alla retta di equazione x=\dfrac{\pi}{2} ha un minimo locale per y=\dfrac{\pi}{2}, infatti con calcoli del tutto analoghi e considerando lo sviluppo al secondo ordine \cos t= 1-\dfrac{1}{2}t^2 + o(t^2) si mostra che

(279)   \begin{equation*} \begin{split} f\left ( \frac{\pi}{2}, \frac{\pi}{2} + t\right ) = h(t) = & \left (1 + \sin \left ( t + \frac{\pi}{2} \right )\right )^2 - 4  - 2\sin \left ( t + \frac{\pi}{2} \right ) \\ = & (1 +\cos t)^2 - 4 - 2\cos t \\ = & \left (1 + 1 - \frac{1}{2}t^2 + o(t^2) \right )^2 - 4 - 2\left (1 - \frac{1}{2}t^2 + o(t^2) \right ) \\ =& 4 - 2t^2 - 4 - 2 + t^2 + o(t^2) \\ = & -2 - t^2\big(1 + o(1) \big), \end{split} \end{equation*}

che quindi ha un massimo locale per t=0, situazione rappresentata in blu in figura 11. Anche in questo caso la stessa conclusione poteva essere dedotta da

(280)   \begin{equation*} \begin{split} f\left ( \frac{\pi}{2}, \frac{\pi}{2} + t\right ) = h(t) = & \left (1 + \sin \left ( t + \frac{\pi}{2} \right )\right )^2 - 4  - 2\sin \left ( t + \frac{\pi}{2} \right ) \\ = & (1 +\cos t)^2 - 4 - 2\cos t \\ = & \cos^2 t -3, \end{split} \end{equation*}

che ha un massimo assoluto in t=0.

Poiché le restrizioni di f a due curve distinte passanti per \left (\dfrac{\pi}{2},\dfrac{\pi}{2} \right ) possiedono rispettivamente un minimo e un massimo locale in corrispondenza di \left (\dfrac{\pi}{2},\dfrac{\pi}{2} \right ), tale punto è di sella.

\bullet \left (\dfrac{\pi}{2},\pi \right ). Come per il punto \left (\frac{\pi}{2},0 \right ), si ha

(281)   \begin{equation*} \nabla^2 f \left (\frac{\pi}{2},\pi\right ) = \begin{pmatrix} 2	&	0 \\ 0	& 2 \end{pmatrix}, \end{equation*}

che è di nuovo definita positiva e, per il teorema 1.10, \left (\dfrac{\pi}{2},\pi \right ) è di minimo relativo per f.

\bullet \left (\dfrac{\pi}{2},\dfrac{3}{2}\pi \right ). La matrice hessiana è pari a

(282)   \begin{equation*} \nabla^2 f \left (\frac{\pi}{2},\frac{3}{2}\pi \right ) = \begin{pmatrix} 4	&	0 \\ 0	& -2 \end{pmatrix} \end{equation*}

che è quindi indefinita poiché i due autovalori 4 e -2 hanno segno opposto. Il teorema 1.10 implica che il punto \left (\dfrac{\pi}{2},\dfrac{3}{2}\pi \right ) è di sella.

\bullet \left (\dfrac{3}{2}\pi,\dfrac{\pi}{2} \right ). La matrice hessiana in questo caso è dunque

(283)   \begin{equation*} \nabla^2 f \left (\frac{3}{2}\pi,\frac{\pi}{2} \right ) = \begin{pmatrix} -4	&	0 \\ 0	& 2 \end{pmatrix} \end{equation*}

anch’essa indefinita. Di nuovo il teorema 1.10 assicura che il punto \left (\dfrac{3}{2}\pi,\dfrac{\pi}{2} \right ) è di sella.

\bullet \left (\dfrac{3}{2}\pi,\dfrac{3}{2}\pi \right ). La matrice hessiana

(284)   \begin{equation*} \nabla^2 f \left (\frac{3}{2}\pi,\frac{3}{2}\pi \right ) = \begin{pmatrix} -8	&	0 \\ 0	& -6 \end{pmatrix} \end{equation*}

è definita negativa in quanto i suoi autovalori sono -8 e -6, entrambi negativi. Pertanto, per il teorema 1.10, il punto considerato è di massimo locale per f.

Osservazione 3.2. Si poteva notare fin da subito dall’espressione di f che

(285)   \begin{equation*} f(x,y) = (\sin x + \sin y)^2 - 4 \sin x - 2 \sin y \leq 4 + 4 + 2 = 10 \qquad \forall (x,y) \in \mathbb{R}^2 \end{equation*}

e che

(286)   \begin{equation*} f \left( \frac{3}{2}\pi +2h\pi, \frac{3}{2}\pi +2k\pi\right ) = 10 \qquad \forall h,k \in \mathbb{Z},  \end{equation*}

quindi i punti \left( \dfrac{3}{2}\pi +2h\pi, \dfrac{3}{2}\pi +2k\pi\right ) sono di massimo assoluto per f. Scrivendo poi la funzione nel seguente modo

(287)   \begin{equation*} \begin{split} f(x,y) = & (\sin x + \sin y)^2 - 2(\sin x + \sin y ) + 1 - 1 - 2 \sin x \\ = & (\sin x + \sin y - 1)^2 - 1 - 2 \sin x \\ \geq & 0 - 1 -2 \qquad \forall (x,y) \in \mathbb{R}^2 \end{split} \end{equation*}

e osservando che

(288)   \begin{equation*} f \left( \frac{\pi}{2} +2h\pi, k\pi\right ) = -3 \qquad \forall h,k \in \mathbb{Z},  \end{equation*}

si ottiene invece che i punti \left( \dfrac{\pi}{2} +2h\pi, k\pi\right ) sono di minimo assoluto per f.

Osservazione 3.3. La funzione esaminata in questo esercizio rappresenta l’energia potenziale di un sistema fisico costituito da due aste in un piano verticale soggette a vincoli forniti da cerniere e molle e a cui sono applicati in maniera solidale dei punti materiali. Si veda la meccanica lagrangiana per la discussione di questo e di altri esercizi dal punto di vista fisico.

In tale sistema fisico le variabili x e y della funzione f rappresentano rispettivamente gli angoli \theta e \beta che le aste formano con l’asse orizzontale.

Lo studio dei punti critici dell’energia potenziale è di notevole importanza in questo genere di applicazioni, in quanto essi rappresentano le configurazioni stazionarie del sistema: se il sistema viene portato in una tale configurazione e lasciato in quiete, rimane in quiete.

Oltre a determinare gli stati stazionari del sistema, è anche necessario classificarli in funzione della loro stabilità: uno stato stazionario è detto stabile se, sottoposto a una piccola perturbazione, il sistema ritorna in tale stato. Si vede che gli stati stazionari stabili sono quelli descritti dai punti di minimo locale per l’energia potenziale.

Queste considerazioni spiegano l’estrema importanza dello studio della natura dei punti critici di una funzione di più variabili nelle applicazioni.


 

Esercizio 16  (\bigstar\bigstar\bigstar\largewhitestar\largewhitestar). Determinare estremi assoluti e locali della funzione f \colon \mathbb{R}^2 \to \mathbb{R} definita da

(289)   \begin{equation*} f(x,y) = x^2 + 4x \cos y + 8 \sin y \qquad \forall (x,y) \in \mathbb{R}^2. \end{equation*}

Svolgimento.

Restringendo la funzione f alla retta di equazione y=0 si ottiene

(290)   \begin{equation*} \lim_{x \to \pm \infty} f(x,0) = \lim_{x \to \pm \infty} (x^2 + 4x) = +\infty, \end{equation*}

da cui

(291)   \begin{equation*} \sup_{\mathbb{R}^2} f = + \infty. \end{equation*}

Dimostriamo ora che f ha minimo assoluto. Infatti, poiché le funzioni \sin e \cos assumono valori compresi tra -1 e 1, si ha

(292)   \begin{equation*} f(x,y) \geq x^2 - 4|x| - 8 = |x|^2 - 4|x| + 4 - 12 = \big( |x| - 2 \big)^2 - 12 \geq 4 \qquad \forall |x| \geq 6, \,\, \forall y \in \mathbb{R}. \end{equation*}

Dato che ad esempio f \left ( 0,0 \right )= 0 < 4 e poiché f è periodica di periodo 2\pi rispetto a y, questo implica che l’estremo inferiore di f su \mathbb{R}^2 va ricercato nell’insieme

(293)   \begin{equation*} D \coloneqq  [-6,6] \times [0,2\pi]. \end{equation*}

Poiché D è compatto e f è continua (è in realtà di classe \mathcal{C}^\infty(\mathbb{R}^2)), il teorema di Weierstrass 1.8 assicura l’esistenza di \min_D f che, per quanto osservato, corrisponde a \min_{\mathbb{R}^2} f. Tale minimo dunque esiste.

Per determinarlo, insieme ad altri punti di estremo relativo per f, si usa il teorema di Fermat 1.9 ed essi si ricercano tra i punti stazionari di f; calcoliamone, a questo fine, le derivate parziali:

(294)   \begin{equation*} \begin{gathered} f_x(x,y) = 2x + 4 \cos y \qquad \forall (x,y) \in \mathbb{R}^2, \\[5pt] f_y(x,y) = -4x \sin y + 8 \cos y \qquad \forall (x,y) \in \mathbb{R}^2. \end{gathered} \end{equation*}

Determiniamo ora i punti stazionari di f:

(295)   \begin{equation*} \begin{split} \nabla f(x,y) = (0,0) \iff & \begin{cases} x + 2 \cos y = 0 \\ -x \sin y + 2 \cos y = 0 \end{cases} \\ \iff & \begin{cases} x + 2 \cos y = 0 \\ x (1+\sin y) = 0 \end{cases} \\ \iff & \begin{cases} \cos y=0 \\[6pt] x=0 \end{cases} \,\,\vee\,\, \begin{cases} x = 0 \\[6pt] \sin y = - 1, \end{cases} \end{split} \end{equation*}

dove nella seconda equivalenza abbiamo sostituito la seconda equazione con la differenza tra la prima e la seconda, mentre nella terza equivalenza abbiamo distinto i casi x=0 e 1+\sin y=0 nella seconda equazione e sostituito nella prima equazione. Si noti che, nell’ultimo sistema \sin y=-1 implica che \cos y=0, che a sua volta produce x=0 nella prima equazione. Gli ultimi due sistemi in (295) hanno rispettivamente come soluzioni i punti

(296)   \begin{equation*} \left (0, \frac{\pi}{2} + k \pi\right )\quad \forall k \in \mathbb{Z}, \qquad \text{e} \qquad \left (0, \frac{3}{2}\pi + 2k \pi\right )\quad \forall k \in \mathbb{Z}. \end{equation*}

Per la periodicità di f rispetto alla variabile y, è sufficiente studiare la natura dei punti critici contenuti nell’insieme

(297)   \begin{equation*} S\coloneqq \{(x,y) \in \mathbb{R}^2 \colon 0 \leq y < 2\pi \}, \end{equation*}

che sono dunque

(298)   \begin{equation*} \left (0, \frac{\pi}{2}\right ), \qquad \left (0, \frac{3}{2}\pi\right ). \end{equation*}

Per stabilirne la natura, si può provare a usare il criterio stabilito dal teorema 1.10 studiando la segnatura della matrice hessiana di f calcolata in questi punti. A tal fine, calcoliamo le derivate seconde di f:

(299)   \begin{equation*} \begin{gathered} f_{xx}(x,y) = 2 \qquad \forall (x,y) \in \mathbb{R}^2, \\[3pt] f_{xy}(x,y) =  f_{yx}(x,y) = -4\sin y \qquad \forall (x,y) \in \mathbb{R}^2, \\[3pt] \begin{split} f_{yy}(x,y) -4x \cos y - 8 \sin y \qquad \forall (x,y) \in \mathbb{R}^2. \end{split} \end{gathered} \end{equation*}

\bullet \left (0,\dfrac{\pi}{2} \right ). Si ha

(300)   \begin{equation*} \nabla^2 f \left (0,\frac{\pi}{2}\right ) = \begin{pmatrix} 2	&	-4 \\ -4	& -8 \end{pmatrix} \quad \Longrightarrow \quad \det \nabla^2 f \left (0,\frac{\pi}{2}\right ) = -16-16 <0, \end{equation*}

da cui segue, per la proposizione 1.13, che \nabla^2 f \left (0,\dfrac{\pi}{2}\right ) è indefinita e dunque, per il teorema 1.10, il punto \left (0,\dfrac{\pi}{2} \right ) è di sella per f.

Ciò poteva anche essere ottenuto osservando che la restrizione di f alla retta orizzontale di equazione y=\dfrac{\pi}{2}, pari a

(301)   \begin{equation*} f\left (x,\frac{\pi}{2} \right ) = x^2+8 \qquad \forall x \in \mathbb{R}, \end{equation*}

ha un punto di minimo in x=0, mentre la restrizione di f alla retta verticale di equazione x=0, data da

(302)   \begin{equation*} f\left (0,y \right ) = 8 \sin y \qquad \forall y \in \mathbb{R}, \end{equation*}

ha un massimo locale in y= \dfrac{\pi}{2}.

\bullet \left (0,\dfrac{3}{2}\pi \right ). La matrice hessiana di f in questo punto è

(303)   \begin{equation*} \nabla^2 f \left (0,\frac{3}{2}\pi\right ) = \begin{pmatrix} 2	&	4 \\ 4	& 8 \end{pmatrix} \quad \Longrightarrow \quad \det \nabla^2 f \left (0,\frac{\pi}{2}\right ) = 16-16 =0. \end{equation*}

Poiché \operatorname{Tr} \nabla^2 f \left (0,\dfrac{3}{2}\pi\right )>0, la proposizione 1.13 implica che \nabla^2 f \left (0,\dfrac{3}{2}\pi\right ) è semidefinita positiva, quindi per il teorema 1.10 il punto è di minimo relativo o di sella per f. Per stabilire se si tratti dell’uno o dell’altro caso, si possono seguire varie strade.

    \[\quad\]

  1. Il metodo forse più immediato in questo caso consiste nel ricordare che si era precedentemente dimostrato che f possiede dei punti di minimo assoluto in \mathbb{R}^2 e quindi in S. Poiché in S vi sono solo due punti stazionari e abbiamo già visto che il punto \left (0,\dfrac{\pi}{2}\right ) è di sella, l’unica possibilità per \left (0,\dfrac{3}{2}\pi\right ) è che sia di minimo assoluto per f.
  2.  

  3. Un modo più diretto per dimostrare che \left (0,\frac{3}{2}\pi\right ) sia di minimo assoluto per f è scrivere

    (304)   \begin{equation*} \begin{split} f(x,y) = & x^2 + 4x \cos y + 4\cos^2 y - 4 \cos^2 y + 8 \sin y \\ = & (x+2\cos y)^2 + 4\sin^2y - 4 + 8 \sin y \\ = & (x+2\cos y)^2 + 4(\sin^2 y + 2 \sin y + 1) - 8 \\ = & (x+2\cos y)^2  + 4(\sin y + 1)^2 - 8 \qquad \forall (x,y) \in \mathbb{R}^2, \end{split} \end{equation*}

    che ha valore minimo pari a -8, ottenuto se e solo se x+2\cos y=0 e \sin y + 1=0, ossia se e solo se y= \dfrac{3}{2}\pi + 2k\pi con k \in \mathbb{Z} e x=0.

Osservazione 3.4. Anche la funzione f di questo esercizio, come quella dell’esercizio 15, corrisponde all’energia potenziale di un sistema fisico studiato in meccanica lagrangiana.


 
 

Riferimenti bibliografici

[1] Abate, M., Geometria. McGraw-Hill (1996).

[2] Acerbi, E. & Buttazzo, G., Secondo corso di Analisi Matematica. Pitagora Editrice (2016).

[3] Fusco, N. & Marcellini, P. & Sbordone, C., Analisi Matematica due. Liguori Editore (1996).

[4] Qui Si Risolve, Il teorema di Fermat.

[5] Qui Si Risolve, Regola della catena.

[6] Tauraso, R., sito web, foglio di esercizi n 3.

[7] Wikipedia, Criterio di Sylvester.

 
 

Tutta la teoria di analisi matematica

Leggi...

  1. Teoria Insiemi
  2. Il metodo della diagonale di Cantor
  3. Logica elementare
  4. Densità dei numeri razionali nei numeri reali
  5. Insiemi Numerici \left(\mathbb{N},\, \mathbb{Z},\, \mathbb{Q}\right)
  6. Il principio di induzione
  7. Gli assiomi di Peano
  8. L’insieme dei numeri reali: costruzione e applicazioni
  9. Concetti Fondamentali della Retta Reale: Sintesi Teorica
  10. Costruzioni alternative di \mathbb{R}
  11. Binomio di Newton
  12. Spazi metrici, un’introduzione
  13. Disuguaglianza di Bernoulli
  14. Disuguaglianza triangolare
  15. Teoria sulle funzioni
  16. Funzioni elementari: algebriche, esponenziali e logaritmiche
  17. Funzioni elementari: trigonometriche e iperboliche
  18. Funzioni goniometriche: la guida essenziale
  19. Teorema di Bolzano-Weierstrass per le successioni
  20. Criterio del rapporto per le successioni
  21. Definizione e proprietà del numero di Nepero
  22. Limite di una successione monotona
  23. Successioni di Cauchy
  24. Il teorema ponte
  25. Teoria sui limiti
  26. Simboli di Landau
  27. Funzioni continue – Teoria
  28. Il teorema di Weierstrass
  29. Il teorema dei valori intermedi
  30. Il teorema della permanenza del segno
  31. Il teorema di Heine-Cantor
  32. Il teorema di esistenza degli zeri
  33. Il metodo di bisezione
  34. Teorema ponte versione per le funzioni continue
  35. Discontinuità di funzioni monotone
  36. Continuità della funzione inversa
  37. Teorema delle contrazioni o Teorema di punto fisso di Banach-Caccioppoli
  38. Teoria sulle derivate
  39. Calcolo delle derivate: la guida pratica
  40. Teoria sulle funzioni convesse
  41. Il teorema di Darboux
  42. I teoremi di de l’Hôpital
  43. Teorema di Fermat
  44. Teoremi di Rolle e Lagrange
  45. Il teorema di Cauchy
  46. Espansione di Taylor: teoria, esempi e applicazioni pratiche
  47. Polinomi di Taylor nei limiti: istruzioni per l’uso
  48. Integrali definiti e indefiniti
  49. Teorema fondamentale del calcolo integrale (approfondimento)
  50. Integrali ricorsivi
  51. Formule del trapezio, rettangolo e Cavalieri-Simpson
  52. Teoria sugli integrali impropri
  53. Funzioni integrali – Teoria
  54. Introduzione ai numeri complessi – Volume 1 (per un corso di ingegneria — versione semplificata)
  55. Introduzione ai numeri complessi – Volume 1 (per un corso di matematica o fisica)
  56. Serie numeriche: la guida completa
  57. Successioni di funzioni – Teoria
  58. Teoremi sulle successioni di funzioni
    1. 58a. Criterio di Cauchy per la convergenza uniforme
    2. 58b. Limite uniforme di funzioni continue
    3. 58c. Passaggio al limite sotto il segno di integrale
    4. 58d. Limite uniforme di funzioni derivabili
    5. 58e. Piccolo teorema del Dini
    6. 58f. Procedura diagonale e teorema di Ascoli-Arzela
  59. Serie di funzioni – Teoria
  60. Serie di potenze – Teoria
  61. Serie di Fourier – Teoria e applicazioni
  62. Integrali multipli — Parte 1 (teoria)
  63. Integrali multipli — Parte 2 (teoria e esercizi misti)
  64. Regola della Catena — Teoria ed esempi.
  65. Jacobiano associato al cambiamento di coordinate sferiche
  66. Guida ai Massimi e Minimi: Tecniche e Teoria nelle Funzioni Multivariabili
  67. Operatore di Laplace o Laplaciano
  68. Teoria equazioni differenziali
  69. Equazione di Eulero
  70. Teoria ed esercizi sulla funzione Gamma di Eulero
  71. Teoria ed esercizi sulla funzione Beta
  72. Approfondimento numeri complessi
  73. Diverse formulazioni dell’assioma di completezza
  74. Numeri di Delannoy centrali
  75. Esercizi avanzati analisi

 
 

Tutte le cartelle di Analisi Matematica

Leggi...

  1. Prerequisiti di Analisi
    1. Ripasso algebra biennio liceo
    2. Ripasso geometria analitica
    3. Ripasso goniometria e trigonometria
    4. Errori tipici da evitare
    5. Insiemi numerici N,Z,Q,R
    6. Funzioni elementari
    7. Logica elementare
    8. Insiemi
  2. Successioni
    1. Teoria sulle Successioni
    2. Estremo superiore e inferiore
    3. Limiti base
    4. Forme indeterminate
    5. Limiti notevoli
    6. Esercizi misti Successioni
    7. Successioni per ricorrenza
  3. Funzioni
    1. Teoria sulle funzioni
    2. Verifica del limite in funzioni
    3. Limite base in funzioni
    4. Forme indeterminate in funzioni
    5. Limiti notevoli in funzioni
    6. Calcolo asintoti
    7. Studio di funzione senza derivate
    8. Dominio di una funzione
    9. Esercizi misti Funzioni
    10. Esercizi misti sui Limiti
  4. Funzioni continue-lipschitziane-holderiane
    1. Teoria sulle Funzioni continue-lipschitziane-holderiane
    2. Continuità delle funzioni
    3. Continuità uniforme
    4. Teorema degli zeri
    5. Esercizi sul teorema di Weierstrass senza l’uso delle derivate
  5. Calcolo differenziale
    1. Derivate
    2. Calcolo delle derivate
    3. Retta tangente nel calcolo differenziale
    4. Punti di non derivabilità nel calcolo differenziale
    5. Esercizi sul teorema di Weierstrass con l’uso delle derivate
    6. Studio di funzione completo nel calcolo differenziale
    7. Esercizi teorici nel calcolo differenziale
    8. Metodo di bisezione
    9. Metodo di Newton
  6. Teoremi del calcolo differenziale
    1. Teoria sui Teoremi del calcolo differenziale
    2. Teorema di Rolle
    3. Teorema di Lagrange
    4. Teorema di Cauchy
    5. Teorema di De L’Hôpital
  7. Calcolo integrale
    1. Integrale di Riemann
    2. Integrali immediati
    3. Integrale di funzione composta
    4. Integrali per sostituzione
    5. Integrali per parti
    6. Integrali di funzione razionale
    7. Calcolo delle aree
    8. Metodo dei rettangoli e dei trapezi
    9. Esercizi Misti Integrali Indefiniti
    10. Esercizi Misti Integrali Definiti
  8. Integrali impropri
    1. Teoria Integrali impropri
    2. Carattere di un integrale improprio
    3. Calcolo di un integrale improprio
  9. Espansione di Taylor
    1. Teoria Espansione di Taylor
    2. Limiti di funzione con Taylor
    3. Limiti di successione con Taylor
    4. Stime del resto
  10. Funzioni integrali (Approfondimento)
    1. Teoria Funzioni integrali (Approfondimento)
    2. Studio di funzione integrale
    3. Limiti con Taylor e De L’Hôpital
    4. Derivazione di integrali parametrici (Tecnica di Feynmann)
  11. Numeri Complessi
    1. Teoria Numeri complessi
    2. Espressioni con i numeri complessi
    3. Radice di un numero complesso
    4. Equazioni con i numeri complessi
    5. Disequazioni con i numeri complessi
    6. Esercizi misti Numeri complessi
  12. Serie numeriche
    1. Teoria Serie numeriche
    2. Esercizi Serie a termini positivi
    3. Esercizi Serie a termini di segno variabile
    4. Esercizi Serie geometriche e telescopiche
  13. Successioni di funzioni
    1. Teoria Successioni di funzioni
    2. Esercizi Successioni di funzioni
  14. Serie di funzioni
    1. Teoria Serie di funzioni
    2. Esercizi Serie di funzioni
  15. Serie di potenze
    1. Teoria Serie di potenze
    2. Esercizi Serie di potenze
  16. Serie di Fourier
    1. Teoria Serie di Fourier
    2. Esercizi Serie di Fourier
  17. Trasformata di Fourier
    1. Teoria Trasformata di Fourier
    2. Esercizi Trasformata di Fourier
  18. Funzioni di più variabili
    1. Teoria Funzioni di più variabili
    2. Massimi e minimi liberi e vincolati
    3. Limiti in due variabili
    4. Integrali doppi
    5. Integrali tripli
    6. Integrali di linea di prima specie
    7. Integrali di linea di seconda specie
    8. Forme differenziali e campi vettoriali
    9. Teorema di Gauss-Green
    10. Integrali di superficie
    11. Flusso di un campo vettoriale
    12. Teorema di Stokes
    13. Teorema della divergenza
    14. Campi solenoidali
    15. Teorema del Dini
  19. Equazioni differenziali lineari e non lineari
    1. Teoria equazioni differenziali lineari e non lineari
    2. Equazioni differenziali lineari e non lineari del primo ordine omogenee
  20. Equazioni differenziali lineari
    1. Del primo ordine non omogenee
    2. Di ordine superiore al primo,a coefficienti costanti,omogenee
    3. Di ordine superiore al primo,a coefficienti costanti,non omogenee
    4. Di Eulero,di Bernoulli,di Clairaut,di Lagrange e di Abel
    5. Non omogenee avente per omogenea associata un’equazione di Eulero
    6. Sistemi di EDO
  21. Equazioni differenziali non lineari
    1. A variabili separabiliO
    2. A secondo membro omogeneo
    3. Del tipo y’=y(ax+by+c)
    4. Del tipo y’=y(ax+by+c)/(a’x+b’y+c’)
    5. Equazioni differenziali esatte
    6. Mancanti delle variabili x e y
    7. Cenni sullo studio di un’assegnata equazione differenziale non lineare
    8. Di Riccati
    9. Cambi di variabile: simmetrie di Lie
  22. Analisi complessa
    1. Fondamenti
    2. Funzioni olomorfe
    3. Integrale di Cauchy e applicazioni
    4. Teorema della curva di Jordan e teorema fondamentale dell’Algebra
    5. Teorema di inversione di Lagrange
    6. Teorema dei Residui
    7. Funzioni meromorfe
    8. Prodotti infiniti e prodotti di Weierstrass
    9. Continuazione analitica e topologia
    10. Teoremi di rigidità di funzioni olomorfe
    11. Trasformata di Mellin
  23. Equazioni alle derivate parziali
    1. Equazioni del primo ordine
    2. Equazioni del secondo ordine lineari
    3. Equazioni non-lineari
    4. Sistemi di PDE
  24. Funzioni speciali
    1. Funzione Gamma di Eulero
    2. Funzioni Beta,Digamma,Trigamma
    3. Integrali ellittici
    4. Funzioni di Bessel
    5. Funzione zeta di Riemann e funzioni L di Dirichlet
    6. Funzione polilogaritmo
    7. Funzioni ipergeometriche
  25. Analisi funzionale
    1. Misura e integrale di Lebesgue
    2. Spazi Lp,teoremi di completezza e compattezza
    3. Spazi di Hilbert,serie e trasformata di Fourier
    4. Teoria e pratica dei polinomi ortogonali
    5. Spazi di Sobolev
  26. Complementi
    1. Curiosità e approfondimenti
    2. Compiti di analisi
    3. Esercizi avanzati analisi
  27. Funzioni Convesse

 
 

Tutti gli esercizi di geometria

In questa sezione vengono raccolti molti altri esercizi che coprono tutti gli argomenti di geometria proposti all’interno del sito con lo scopo di offrire al lettore la possibilità di approfondire e rinforzare le proprie competenze inerenti a tali argomenti.

Strutture algebriche.





 
 

Risorse didattiche aggiuntive per approfondire la matematica

Leggi...

  • Math Stack Exchange – Parte della rete Stack Exchange, questo sito è un forum di domande e risposte specificamente dedicato alla matematica. È una delle piattaforme più popolari per discutere e risolvere problemi matematici di vario livello, dall’elementare all’avanzato.
  • Art of Problem Solving (AoPS) – Questo sito è molto noto tra gli studenti di matematica di livello avanzato e i partecipanti a competizioni matematiche. Offre forum, corsi online, e risorse educative su una vasta gamma di argomenti.
  • MathOverflow – Questo sito è destinato a matematici professionisti e ricercatori. È una piattaforma per domande di ricerca avanzata in matematica. È strettamente legato a Math Stack Exchange ma è orientato a un pubblico con una formazione più avanzata.
  • PlanetMath – Una comunità collaborativa di matematici che crea e cura articoli enciclopedici e altre risorse di matematica. È simile a Wikipedia, ma focalizzata esclusivamente sulla matematica.
  • Wolfram MathWorld – Una delle risorse online più complete per la matematica. Contiene migliaia di articoli su argomenti di matematica, creati e curati da esperti. Sebbene non sia un forum, è una risorsa eccellente per la teoria matematica.
  • The Math Forum – Un sito storico che offre un’ampia gamma di risorse, inclusi forum di discussione, articoli e risorse educative. Sebbene alcune parti del sito siano state integrate con altri servizi, come NCTM, rimane una risorsa preziosa per la comunità educativa.
  • Stack Overflow (sezione matematica) – Sebbene Stack Overflow sia principalmente noto per la programmazione, ci sono anche discussioni rilevanti di matematica applicata, specialmente nel contesto della scienza dei dati, statistica, e algoritmi.
  • Reddit (r/Math) – Un subreddit popolare dove si possono trovare discussioni su una vasta gamma di argomenti matematici. È meno formale rispetto ai siti di domande e risposte come Math Stack Exchange, ma ha una comunità attiva e molte discussioni interessanti.
  • Brilliant.org – Offre corsi interattivi e problemi di matematica e scienza. È particolarmente utile per chi vuole allenare le proprie capacità di problem solving in matematica.
  • Khan Academy – Una risorsa educativa globale con lezioni video, esercizi interattivi e articoli su una vasta gamma di argomenti di matematica, dalla scuola elementare all’università.






Document









Document